[obm-l] Diferencial de uma função de uma variável

2023-06-10 Por tôpico Pacini Bores
Olá pessoal,
O colega BobRoy me pediu para enviar para vocês  a seguinte dúvida, já que
ele não está conseguindo enviar mensagens para a lista :


A notação de leibniz para f´´(x) = d^2(f) / dx^2 é apenas uma notação ? ou
podemos isolar os numeradores? Já que em   f`(x) =dy/dx  podemos
multiplicar...
Vejo em alguns livros colocando dx^2 como (dx)^2..


Abraços
Pacini

-- 
Esta mensagem foi verificada pelo sistema de antiv�rus e
 acredita-se estar livre de perigo.



[obm-l] Fwd: Diferencial de ordem superior

2023-06-10 Por tôpico Bob Roy
-- Forwarded message -
De: Bob Roy 
Date: qui., 8 de jun. de 2023 às 08:50
Subject: Diferencial de ordem superior
To: 


Olá pessoal,
A notação de leibniz para f´´(x) = d^2(f) / dx^2 é apenas uma notação ? ou
podemos isolar os numeradores?
Vejo em alguns livros colocando dx^2 como (dx)^2..

Bob Roy

-- 
Esta mensagem foi verificada pelo sistema de antiv�rus e
 acredita-se estar livre de perigo.



[obm-l] Diferencial de ordem superior

2023-06-08 Por tôpico Bob Roy
Olá pessoal,
A notação de leibniz para f´´(x) = d^2(f) / dx^2 é apenas uma notação ? ou
podemos isolar os numeradores?
Vejo em alguns livros colocando dx^2 como (dx)^2..

Bob Roy

-- 
Esta mensagem foi verificada pelo sistema de antiv�rus e
 acredita-se estar livre de perigo.



Re: [obm-l] Irracionalidade de pi

2023-05-26 Por tôpico jose rodrigo
o famoso complexo de gênio.


On Sat, May 27, 2023 at 12:42 AM Anderson Torres <
torres.anderson...@gmail.com> wrote:

>
>
> Em sex, 26 de mai de 2023 18:25, Israel Meireles Chrisostomo <
> israelmchrisost...@gmail.com> escreveu:
>
>>
>> eu quero uma audiência com um matemático profissional, eu acabei de
>> provar a irracionalidade de pi. alguém com tempo para corrigir?
>>
>
>
> O que tem de especial nisso para desejar um matemático profissional?
>
> --
>> Israel Meireles Chrisostomo
>>
>> --
>> Esta mensagem foi verificada pelo sistema de antivírus e
>> acredita-se estar livre de perigo.
>
>
> --
> Esta mensagem foi verificada pelo sistema de antivírus e
> acredita-se estar livre de perigo.

-- 
Esta mensagem foi verificada pelo sistema de antiv�rus e
 acredita-se estar livre de perigo.



Re: [obm-l] Irracionalidade de pi

2023-05-26 Por tôpico Anderson Torres
Em sex, 26 de mai de 2023 18:25, Israel Meireles Chrisostomo <
israelmchrisost...@gmail.com> escreveu:

>
> eu quero uma audiência com um matemático profissional, eu acabei de provar
> a irracionalidade de pi. alguém com tempo para corrigir?
>


O que tem de especial nisso para desejar um matemático profissional?

-- 
> Israel Meireles Chrisostomo
>
> --
> Esta mensagem foi verificada pelo sistema de antivírus e
> acredita-se estar livre de perigo.

-- 
Esta mensagem foi verificada pelo sistema de antiv�rus e
 acredita-se estar livre de perigo.



Re: [obm-l] Irracionalidade de pi

2023-05-26 Por tôpico Desire Yema
você pode mandar a prova qui assim todo mundo pode ver e criticar

Le ven. 26 mai 2023 à 18:25, Israel Meireles Chrisostomo
 a écrit :
>
>
> eu quero uma audiência com um matemático profissional, eu acabei de provar a 
> irracionalidade de pi. alguém com tempo para corrigir?
> --
> Israel Meireles Chrisostomo
>
> --
> Esta mensagem foi verificada pelo sistema de antivírus e
> acredita-se estar livre de perigo.

-- 
Esta mensagem foi verificada pelo sistema de antiv�rus e
 acredita-se estar livre de perigo.


=
Instru��es para entrar na lista, sair da lista e usar a lista em
http://www.mat.puc-rio.br/~obmlistas/obm-l.html
=


[obm-l] Irracionalidade de pi

2023-05-26 Por tôpico Israel Meireles Chrisostomo
eu quero uma audiência com um matemático profissional, eu acabei de provar
a irracionalidade de pi. alguém com tempo para corrigir?
-- 
Israel Meireles Chrisostomo

-- 
Esta mensagem foi verificada pelo sistema de antiv�rus e
 acredita-se estar livre de perigo.



Re: [obm-l] Propriedade do no 7

2023-05-12 Por tôpico Claudio Buffara
Iniciando uma discussão sobre pedagogia...

Quem acha que esse tipo de problema deveria fazer parte do currículo de
álgebra na escola, digamos a partir do 8o ou 9o ano?
Há vários outros na mesma linha...
1) Ache todas as triplas de primos trigêmeos (triplas de naturais da forma
(n,n+2,n+4) em que todos são primos);
2) O quadrado de um inteiro ímpar excede em 1 algum múltiplo de 8;
3) Ache todos os primos da forma n^4 + 4^m
4) Ache todos os naturais que podem ser expressos como uma diferença de
quadrados de naturais.  Pra quais naturais essa expressão é única?
etc...

Problemas do tipo "ache todos" ou "prove que não existe"  são interessantes
porque mostram pra garotada que nem  todo problema tem uma solução única e
que provar que um dado problema não tem solução também é, de certa forma,
uma solução.

[]s,
Claudio.


On Thu, May 11, 2023 at 10:57 AM Caio Costa  wrote:

> N = n³-1 = (n-1)*(n² + n + 1).
> n-1 divide n³ - 1, logo se n³ -1 é primo, então n-1 = 1, daí n = 2 e N = 7.
>
> Em qui., 11 de mai. de 2023 às 11:23, Luiz Alberto Salomao <
> ladsalo...@gmail.com> escreveu:
>
>> Olá, Artur
>> Cultura sempre é útil. Muito bacana!
>> Você conhece alguma prova desse resultado?
>> Luiz Alberto.
>>
>> Em qui., 11 de mai. de 2023 às 08:20, Artur Costa Steiner <
>> artur.costa.stei...@gmail.com> escreveu:
>>
>>> 7 é o único primo seguido por um cubo. Alguns talvez achem isso uma
>>> curiosidade interessante. Outros talvez achem cultura inútil.rsss
>>>
>>> Artur
>>>
>>> --
>>> Esta mensagem foi verificada pelo sistema de antivírus e
>>> acredita-se estar livre de perigo.
>>
>>
>> --
>> Esta mensagem foi verificada pelo sistema de antivírus e
>> acredita-se estar livre de perigo.
>
>
> --
> Esta mensagem foi verificada pelo sistema de antivírus e
> acredita-se estar livre de perigo.

-- 
Esta mensagem foi verificada pelo sistema de antiv�rus e
 acredita-se estar livre de perigo.



Re: [obm-l] Propriedade do no 7

2023-05-11 Por tôpico Artur Costa Steiner
O Caio apresentou a prova.
Artur

Em qui., 11 de mai. de 2023 11:57, Caio Costa 
escreveu:

> N = n³-1 = (n-1)*(n² + n + 1).
> n-1 divide n³ - 1, logo se n³ -1 é primo, então n-1 = 1, daí n = 2 e N = 7.
>
> Em qui., 11 de mai. de 2023 às 11:23, Luiz Alberto Salomao <
> ladsalo...@gmail.com> escreveu:
>
>> Olá, Artur
>> Cultura sempre é útil. Muito bacana!
>> Você conhece alguma prova desse resultado?
>> Luiz Alberto.
>>
>> Em qui., 11 de mai. de 2023 às 08:20, Artur Costa Steiner <
>> artur.costa.stei...@gmail.com> escreveu:
>>
>>> 7 é o único primo seguido por um cubo. Alguns talvez achem isso uma
>>> curiosidade interessante. Outros talvez achem cultura inútil.rsss
>>>
>>> Artur
>>>
>>> --
>>> Esta mensagem foi verificada pelo sistema de antivírus e
>>> acredita-se estar livre de perigo.
>>
>>
>> --
>> Esta mensagem foi verificada pelo sistema de antivírus e
>> acredita-se estar livre de perigo.
>
>
> --
> Esta mensagem foi verificada pelo sistema de antivírus e
> acredita-se estar livre de perigo.

-- 
Esta mensagem foi verificada pelo sistema de antiv�rus e
 acredita-se estar livre de perigo.



Re: [obm-l] Propriedade do no 7

2023-05-11 Por tôpico Caio Costa
N = n³-1 = (n-1)*(n² + n + 1).
n-1 divide n³ - 1, logo se n³ -1 é primo, então n-1 = 1, daí n = 2 e N = 7.

Em qui., 11 de mai. de 2023 às 11:23, Luiz Alberto Salomao <
ladsalo...@gmail.com> escreveu:

> Olá, Artur
> Cultura sempre é útil. Muito bacana!
> Você conhece alguma prova desse resultado?
> Luiz Alberto.
>
> Em qui., 11 de mai. de 2023 às 08:20, Artur Costa Steiner <
> artur.costa.stei...@gmail.com> escreveu:
>
>> 7 é o único primo seguido por um cubo. Alguns talvez achem isso uma
>> curiosidade interessante. Outros talvez achem cultura inútil.rsss
>>
>> Artur
>>
>> --
>> Esta mensagem foi verificada pelo sistema de antivírus e
>> acredita-se estar livre de perigo.
>
>
> --
> Esta mensagem foi verificada pelo sistema de antivírus e
> acredita-se estar livre de perigo.

-- 
Esta mensagem foi verificada pelo sistema de antiv�rus e
 acredita-se estar livre de perigo.



Re: [obm-l] Propriedade do no 7

2023-05-11 Por tôpico Luiz Alberto Salomao
Olá, Artur
Cultura sempre é útil. Muito bacana!
Você conhece alguma prova desse resultado?
Luiz Alberto.

Em qui., 11 de mai. de 2023 às 08:20, Artur Costa Steiner <
artur.costa.stei...@gmail.com> escreveu:

> 7 é o único primo seguido por um cubo. Alguns talvez achem isso uma
> curiosidade interessante. Outros talvez achem cultura inútil.rsss
>
> Artur
>
> --
> Esta mensagem foi verificada pelo sistema de antivírus e
> acredita-se estar livre de perigo.

-- 
Esta mensagem foi verificada pelo sistema de antiv�rus e
 acredita-se estar livre de perigo.



[obm-l] Propriedade do no 7

2023-05-11 Por tôpico Artur Costa Steiner
7 é o único primo seguido por um cubo. Alguns talvez achem isso uma
curiosidade interessante. Outros talvez achem cultura inútil.rsss

Artur

-- 
Esta mensagem foi verificada pelo sistema de antiv�rus e
 acredita-se estar livre de perigo.



[obm-l] Re: [obm-l] Re: [obm-l] Uma recorrência diferente

2023-04-06 Por tôpico Esdras Muniz
Vc pode pegar a função geratriz e usar a fórmula de Ramanujan pra calcular
o termo geral. Acredito que a função geratriz seja: e^{x^2/2+x}, já na
forma (x_n)x^n/n!...

Em qui, 6 de abr de 2023 19:03, Carlos Gustavo Tamm de Araujo Moreira <
g...@impa.br> escreveu:

> Caro Vanderlei,
> Não parece haver uma fórmula fechada muito simples. Veja
> https://oeis.org/A85 para várias referências
> sobre essa sequência.
> Abraços,
> Gugu
>
> On Wed, Apr 5, 2023 at 11:41 PM Professor Vanderlei Nemitz <
> vanderma...@gmail.com> wrote:
>
>> Oi, mestres!
>>
>> Estava resolvendo um problema de combinatória e obtive essa recorrência:
>>
>> *x(n) = x(n - 1) + (n - 1).x(n - 2), com x1 = 1 e x2 = 2*.
>>
>> Por exemplo, x3 = x2 + 2.x1 e x9 = x8 + 8.x7
>>
>> Como resolver quando os coeficientes não são todos constantes?
>>
>> Apenas como curiosidade, o problema que originou a recorrência é:
>> (IME - RJ) - Um professor dá um teste surpresa para uma turma de 9
>> alunos, e diz que o teste pode ser feito sozinho ou em grupos de 2 alunos.
>> De quantas formas a turma pode ser organizar para fazer o teste? (Por
>> exemplo, uma turma de 3 alunos pode ser organizar de 4 formas e uma turma
>> de 4 alunos pode se organizar de 10 formas)
>>
>>
>>
>> 
>>  Não
>> contém vírus.www.avast.com
>> 
>> <#m_-6699161896707809793_m_4118911927943204904_DAB4FAD8-2DD7-40BB-A1B8-4E2AA1F9FDF2>
>>
>> --
>> Esta mensagem foi verificada pelo sistema de antivírus e
>> acredita-se estar livre de perigo.
>
>
> --
> Esta mensagem foi verificada pelo sistema de antivírus e
> acredita-se estar livre de perigo.

-- 
Esta mensagem foi verificada pelo sistema de antiv�rus e
 acredita-se estar livre de perigo.



[obm-l] Re: [obm-l] Uma recorrência diferente

2023-04-06 Por tôpico Carlos Gustavo Tamm de Araujo Moreira
Caro Vanderlei,
Não parece haver uma fórmula fechada muito simples. Veja
https://oeis.org/A85 para várias referências
sobre essa sequência.
Abraços,
Gugu

On Wed, Apr 5, 2023 at 11:41 PM Professor Vanderlei Nemitz <
vanderma...@gmail.com> wrote:

> Oi, mestres!
>
> Estava resolvendo um problema de combinatória e obtive essa recorrência:
>
> *x(n) = x(n - 1) + (n - 1).x(n - 2), com x1 = 1 e x2 = 2*.
>
> Por exemplo, x3 = x2 + 2.x1 e x9 = x8 + 8.x7
>
> Como resolver quando os coeficientes não são todos constantes?
>
> Apenas como curiosidade, o problema que originou a recorrência é:
> (IME - RJ) - Um professor dá um teste surpresa para uma turma de 9 alunos,
> e diz que o teste pode ser feito sozinho ou em grupos de 2 alunos. De
> quantas formas a turma pode ser organizar para fazer o teste? (Por exemplo,
> uma turma de 3 alunos pode ser organizar de 4 formas e uma turma de 4
> alunos pode se organizar de 10 formas)
>
>
>
> 
>  Não
> contém vírus.www.avast.com
> 
> <#m_4118911927943204904_DAB4FAD8-2DD7-40BB-A1B8-4E2AA1F9FDF2>
>
> --
> Esta mensagem foi verificada pelo sistema de antivírus e
> acredita-se estar livre de perigo.

-- 
Esta mensagem foi verificada pelo sistema de antiv�rus e
 acredita-se estar livre de perigo.



[obm-l] Re: [obm-l] Uma recorrência diferente

2023-04-06 Por tôpico Anderson Torres
Em qua., 5 de abr. de 2023 às 23:40, Professor Vanderlei Nemitz <
vanderma...@gmail.com> escreveu:

> Oi, mestres!
>
> Estava resolvendo um problema de combinatória e obtive essa recorrência:
>
> *x(n) = x(n - 1) + (n - 1).x(n - 2), com x1 = 1 e x2 = 2*.
>
> Por exemplo, x3 = x2 + 2.x1 e x9 = x8 + 8.x7
>
> Como resolver quando os coeficientes não são todos constantes?
>

Nem sempre dá para esperar que uma recorrência seja "resolvível".

Você pode ir tabelando e graficando alguns valores para ver se surge alguma
coisa, e daí imaginar alguma fórmula, algo como "bem, isso aqui parece um
gráfico de um fatorial"...

E, na real, nem sempre é tão útil ter uma fórmula pronta. Um computador
pode muito bem calcular os termos dessa sequência, em um processo idêntico
ao que um ser humano faria só que mais rápido (e com menos chance de
errar). E o vestibular não exigiu calcular isso para n=100, mas apenas para
n=9.

Todavia, é uma boa fuçar. Pensei em algo como usar uma sequência auxiliar,
y(n)=x(n+1)/x(n). Isso nos daria uma outra recorrência, a saber,

y(n)=1+(n/y(n-1))

O que estranhamente gera uma espécie de fração contínua. Mas não consigo
imaginar algo muito melhor que isso.



>
> Apenas como curiosidade, o problema que originou a recorrência é:
> (IME - RJ) - Um professor dá um teste surpresa para uma turma de 9 alunos,
> e diz que o teste pode ser feito sozinho ou em grupos de 2 alunos. De
> quantas formas a turma pode ser organizar para fazer o teste? (Por exemplo,
> uma turma de 3 alunos pode ser organizar de 4 formas e uma turma de 4
> alunos pode se organizar de 10 formas)
>

>
>
> 
>  Não
> contém vírus.www.avast.com
> 
> <#m_-8406560445346393254_DAB4FAD8-2DD7-40BB-A1B8-4E2AA1F9FDF2>
>
> --
> Esta mensagem foi verificada pelo sistema de antivírus e
> acredita-se estar livre de perigo.

-- 
Esta mensagem foi verificada pelo sistema de antiv�rus e
 acredita-se estar livre de perigo.



[obm-l] Uma recorrência diferente

2023-04-05 Por tôpico Professor Vanderlei Nemitz
Oi, mestres!

Estava resolvendo um problema de combinatória e obtive essa recorrência:

*x(n) = x(n - 1) + (n - 1).x(n - 2), com x1 = 1 e x2 = 2*.

Por exemplo, x3 = x2 + 2.x1 e x9 = x8 + 8.x7

Como resolver quando os coeficientes não são todos constantes?

Apenas como curiosidade, o problema que originou a recorrência é:
(IME - RJ) - Um professor dá um teste surpresa para uma turma de 9 alunos,
e diz que o teste pode ser feito sozinho ou em grupos de 2 alunos. De
quantas formas a turma pode ser organizar para fazer o teste? (Por exemplo,
uma turma de 3 alunos pode ser organizar de 4 formas e uma turma de 4
alunos pode se organizar de 10 formas)



Não
contém vírus.www.avast.com

<#DAB4FAD8-2DD7-40BB-A1B8-4E2AA1F9FDF2>

-- 
Esta mensagem foi verificada pelo sistema de antiv�rus e
 acredita-se estar livre de perigo.



Re: [obm-l] Cone Sul

2023-03-18 Por tôpico Anderson Torres
Em seg., 13 de mar. de 2023 às 10:42, Armando Staib
 escreveu:
>
> Rsse repositorio é PAGO  certo!?

Não.

>
> Em seg, 13 de mar de 2023 10:26, Ian Barquette  
> escreveu:
>>
>> O repositório da "Art of Problem Solving" é muito completo, porém as 
>> questões são em inglês
>>
>> Em seg., 13 de mar. de 2023 09:09, Pedro Júnior 
>>  escreveu:
>>>
>>> Olá pessoal, muito bom dia.
>>> Gostaria de saber se tem um site oficial da competição "Cone Sul de 
>>> Matemática"? Procurei o banco de provas pelo Google e não encontrei. Me 
>>> remete ao site da OBM e também não vi por lá.
>>>
>>> Desde já fico grato.
>>>
>>> --
>>> Esta mensagem foi verificada pelo sistema de antivírus e
>>> acredita-se estar livre de perigo.
>>
>>
>> --
>> Esta mensagem foi verificada pelo sistema de antivírus e
>> acredita-se estar livre de perigo.
>
>
> --
> Esta mensagem foi verificada pelo sistema de antivírus e
> acredita-se estar livre de perigo.

-- 
Esta mensagem foi verificada pelo sistema de antiv�rus e
 acredita-se estar livre de perigo.


=
Instru��es para entrar na lista, sair da lista e usar a lista em
http://www.mat.puc-rio.br/~obmlistas/obm-l.html
=


Re: [obm-l] Cone Sul

2023-03-13 Por tôpico Pazó 0192
No site https://sites.google.com/site/selecaoconesul/ você encontrará todo
material para treinamento e os testes da seletiva da Cone Sul, além de
várias outras informações.

On Mon, Mar 13, 2023 at 9:09 AM Pedro Júnior 
wrote:

> Olá pessoal, muito bom dia.
> Gostaria de saber se tem um site oficial da competição "Cone Sul de
> Matemática"? Procurei o banco de provas pelo Google e não encontrei. Me
> remete ao site da OBM e também não vi por lá.
>
> Desde já fico grato.
>
> --
> Esta mensagem foi verificada pelo sistema de antivírus e
> acredita-se estar livre de perigo.

-- 
Esta mensagem foi verificada pelo sistema de antiv�rus e
 acredita-se estar livre de perigo.



Re: [obm-l] Cone Sul

2023-03-13 Por tôpico João
Também tem o site do treinamento da cone sul do brasil, com listas e testes
de seleção
https://sites.google.com/site/selecaoconesul/


On Mon, 13 Mar 2023 at 10:26 Ian Barquette 
wrote:

> O repositório da "Art of Problem Solving" é muito completo, porém as
> questões são em inglês
>
> Em seg., 13 de mar. de 2023 09:09, Pedro Júnior <
> pedromatematic...@gmail.com> escreveu:
>
>> Olá pessoal, muito bom dia.
>> Gostaria de saber se tem um site oficial da competição "Cone Sul de
>> Matemática"? Procurei o banco de provas pelo Google e não encontrei. Me
>> remete ao site da OBM e também não vi por lá.
>>
>> Desde já fico grato.
>>
>> --
>> Esta mensagem foi verificada pelo sistema de antivírus e
>> acredita-se estar livre de perigo.
>
>
> --
> Esta mensagem foi verificada pelo sistema de antivírus e
> acredita-se estar livre de perigo.

-- 
Esta mensagem foi verificada pelo sistema de antiv�rus e
 acredita-se estar livre de perigo.



Re: [obm-l] Cone Sul

2023-03-13 Por tôpico Armando Staib
Rsse repositorio é PAGO  certo!?

Em seg, 13 de mar de 2023 10:26, Ian Barquette 
escreveu:

> O repositório da "Art of Problem Solving" é muito completo, porém as
> questões são em inglês
>
> Em seg., 13 de mar. de 2023 09:09, Pedro Júnior <
> pedromatematic...@gmail.com> escreveu:
>
>> Olá pessoal, muito bom dia.
>> Gostaria de saber se tem um site oficial da competição "Cone Sul de
>> Matemática"? Procurei o banco de provas pelo Google e não encontrei. Me
>> remete ao site da OBM e também não vi por lá.
>>
>> Desde já fico grato.
>>
>> --
>> Esta mensagem foi verificada pelo sistema de antivírus e
>> acredita-se estar livre de perigo.
>
>
> --
> Esta mensagem foi verificada pelo sistema de antivírus e
> acredita-se estar livre de perigo.

-- 
Esta mensagem foi verificada pelo sistema de antiv�rus e
 acredita-se estar livre de perigo.



Re: [obm-l] Cone Sul

2023-03-13 Por tôpico Ian Barquette
O repositório da "Art of Problem Solving" é muito completo, porém as
questões são em inglês

Em seg., 13 de mar. de 2023 09:09, Pedro Júnior 
escreveu:

> Olá pessoal, muito bom dia.
> Gostaria de saber se tem um site oficial da competição "Cone Sul de
> Matemática"? Procurei o banco de provas pelo Google e não encontrei. Me
> remete ao site da OBM e também não vi por lá.
>
> Desde já fico grato.
>
> --
> Esta mensagem foi verificada pelo sistema de antivírus e
> acredita-se estar livre de perigo.

-- 
Esta mensagem foi verificada pelo sistema de antiv�rus e
 acredita-se estar livre de perigo.



Re: [obm-l] Cone Sul

2023-03-13 Por tôpico Anderson Torres
Em seg, 13 de mar de 2023 09:09, Pedro Júnior 
escreveu:

> Olá pessoal, muito bom dia.
> Gostaria de saber se tem um site oficial da competição "Cone Sul de
> Matemática"? Procurei o banco de provas pelo Google e não encontrei. Me
> remete ao site da OBM e também não vi por lá.
>

Já tentou o Mathlinks? Se você só está à procura dos enunciados, lá tem.


> Desde já fico grato.
>
> --
> Esta mensagem foi verificada pelo sistema de antivírus e
> acredita-se estar livre de perigo.

-- 
Esta mensagem foi verificada pelo sistema de antiv�rus e
 acredita-se estar livre de perigo.



Re: [obm-l] Cone Sul

2023-03-13 Por tôpico Arthur Vieira
https://www.obm.org.br/como-se-preparar/provas-e-gabaritos/

Tem que descer um pouco, mas tem as provas dessa e de outras competições.

Em seg., 13 de mar. de 2023 às 09:09, Pedro Júnior <
pedromatematic...@gmail.com> escreveu:

> Olá pessoal, muito bom dia.
> Gostaria de saber se tem um site oficial da competição "Cone Sul de
> Matemática"? Procurei o banco de provas pelo Google e não encontrei. Me
> remete ao site da OBM e também não vi por lá.
>
> Desde já fico grato.
>
> --
> Esta mensagem foi verificada pelo sistema de antivírus e
> acredita-se estar livre de perigo.

-- 
Esta mensagem foi verificada pelo sistema de antiv�rus e
 acredita-se estar livre de perigo.



[obm-l] Cone Sul

2023-03-13 Por tôpico Pedro Júnior
Olá pessoal, muito bom dia.
Gostaria de saber se tem um site oficial da competição "Cone Sul de
Matemática"? Procurei o banco de provas pelo Google e não encontrei. Me
remete ao site da OBM e também não vi por lá.

Desde já fico grato.

-- 
Esta mensagem foi verificada pelo sistema de antiv�rus e
 acredita-se estar livre de perigo.



[obm-l] Re: [obm-l] Seria por distribuição binomial ou alguma recorrência

2023-02-28 Por tôpico Anderson Torres
Em ter, 28 de fev de 2023 11:52, Bianca Flores 
escreveu:

> Alguém poderia ajudar com essa questão: estou frustrada porque não consigo
> chegar ao gabarito E.
>
> Um estudante preenche, aleatoriamente e de forma independente cada uma das
> questões, um exame de múltipla escolha com 5 respostas possíveis (das quais
> apenas uma é correta) para cada uma de 25 questões. A probabilidade que ele
> acerte um número par de questões é dada por:
>
> (A)(1-(4/5)^25)/2
> (B)(1-(3/5)^25)/2
> (C)((3/5)^25)/2
> (D)(1+(4/5)^25)/2
> (E)(1+(3/5)^25)/2
>

Bem, a probabilidade de acertar k questões de n é C(n,k) (1/5)^k
(4/5)^(n-k).
Soma isso para todo k par (lá ele) e vê no que dá.


>
> Tento de todas as formar usar a distribuição binomial, alguma recorrência,
> mas sem sucesso.
> Bianca
>
> --
> Esta mensagem foi verificada pelo sistema de antivírus e
> acredita-se estar livre de perigo.
>

-- 
Esta mensagem foi verificada pelo sistema de antiv�rus e
 acredita-se estar livre de perigo.



[obm-l] Re: [obm-l] Seria por distribuição binomial ou alguma recorrência

2023-02-28 Por tôpico joao pedro b menezes
A probabilidade do estudante acertar um número n de questões é [ (1/5)^n *
(4/5)^(25-n) ] * n!*(25-n)!/25! . ( o primeiro segmento, separo por [
...], indica a probabilidade de ele acertar n questões em uma ordem
definida, enquanto a segunda parte se refere ao número de combinações
possíveis em que ele acerta n questões ) . Agora é somar e fatorar

On Tue, Feb 28, 2023, 11:52 Bianca Flores  wrote:

> Alguém poderia ajudar com essa questão: estou frustrada porque não consigo
> chegar ao gabarito E.
>
> Um estudante preenche, aleatoriamente e de forma independente cada uma das
> questões, um exame de múltipla escolha com 5 respostas possíveis (das quais
> apenas uma é correta) para cada uma de 25 questões. A probabilidade que ele
> acerte um número par de questões é dada por:
>
> (A)(1-(4/5)^25)/2
> (B)(1-(3/5)^25)/2
> (C)((3/5)^25)/2
> (D)(1+(4/5)^25)/2
> (E)(1+(3/5)^25)/2
>
> Tento de todas as formar usar a distribuição binomial, alguma recorrência,
> mas sem sucesso.
> Bianca
>
> --
> Esta mensagem foi verificada pelo sistema de antivírus e
> acredita-se estar livre de perigo.
>

-- 
Esta mensagem foi verificada pelo sistema de antiv�rus e
 acredita-se estar livre de perigo.



[obm-l] Re: [obm-l] Seria por distribuição binomial ou alguma recorrência

2023-02-28 Por tôpico Esdras Muniz
Letra E na verdade 

Em ter., 28 de fev. de 2023 às 11:52, Bianca Flores 
escreveu:

> Alguém poderia ajudar com essa questão: estou frustrada porque não consigo
> chegar ao gabarito E.
>
> Um estudante preenche, aleatoriamente e de forma independente cada uma das
> questões, um exame de múltipla escolha com 5 respostas possíveis (das quais
> apenas uma é correta) para cada uma de 25 questões. A probabilidade que ele
> acerte um número par de questões é dada por:
>
> (A)(1-(4/5)^25)/2
> (B)(1-(3/5)^25)/2
> (C)((3/5)^25)/2
> (D)(1+(4/5)^25)/2
> (E)(1+(3/5)^25)/2
>
> Tento de todas as formar usar a distribuição binomial, alguma recorrência,
> mas sem sucesso.
> Bianca
>
> --
> Esta mensagem foi verificada pelo sistema de antivírus e
> acredita-se estar livre de perigo.
>
-- 
Esdras Muniz Mota
Mestrando em Matemática
Universidade Federal do Ceará

-- 
Esta mensagem foi verificada pelo sistema de antiv�rus e
 acredita-se estar livre de perigo.



[obm-l] Re: [obm-l] Seria por distribuição binomial ou alguma recorrência

2023-02-28 Por tôpico Esdras Muniz
Veja 1 como a soma de todas as probabilidades possíveis e (3/5)^25 como a
probabilidade de ele acertas uma quantidade par menos a probabilidade de
ele acertar uma quantidade ímpar.

Em ter., 28 de fev. de 2023 às 11:58, Esdras Muniz <
esdrasmunizm...@gmail.com> escreveu:

> Acredito que é letra B. Vc pode fazer usando binômio de Newton…
>
> Em ter., 28 de fev. de 2023 às 11:52, Bianca Flores 
> escreveu:
>
>> Alguém poderia ajudar com essa questão: estou frustrada porque não
>> consigo chegar ao gabarito E.
>>
>> Um estudante preenche, aleatoriamente e de forma independente cada uma
>> das questões, um exame de múltipla escolha com 5 respostas possíveis (das
>> quais apenas uma é correta) para cada uma de 25 questões. A probabilidade
>> que ele acerte um número par de questões é dada por:
>>
>> (A)(1-(4/5)^25)/2
>> (B)(1-(3/5)^25)/2
>> (C)((3/5)^25)/2
>> (D)(1+(4/5)^25)/2
>> (E)(1+(3/5)^25)/2
>>
>> Tento de todas as formar usar a distribuição binomial, alguma
>> recorrência, mas sem sucesso.
>> Bianca
>>
>> --
>> Esta mensagem foi verificada pelo sistema de antivírus e
>> acredita-se estar livre de perigo.
>>
> --
> Esdras Muniz Mota
> Mestrando em Matemática
> Universidade Federal do Ceará
>
>
> --
Esdras Muniz Mota
Mestrando em Matemática
Universidade Federal do Ceará

-- 
Esta mensagem foi verificada pelo sistema de antiv�rus e
 acredita-se estar livre de perigo.



[obm-l] Re: [obm-l] Seria por distribuição binomial ou alguma recorrência

2023-02-28 Por tôpico Esdras Muniz
Acredito que é letra B. Vc pode fazer usando binômio de Newton…

Em ter., 28 de fev. de 2023 às 11:52, Bianca Flores 
escreveu:

> Alguém poderia ajudar com essa questão: estou frustrada porque não consigo
> chegar ao gabarito E.
>
> Um estudante preenche, aleatoriamente e de forma independente cada uma das
> questões, um exame de múltipla escolha com 5 respostas possíveis (das quais
> apenas uma é correta) para cada uma de 25 questões. A probabilidade que ele
> acerte um número par de questões é dada por:
>
> (A)(1-(4/5)^25)/2
> (B)(1-(3/5)^25)/2
> (C)((3/5)^25)/2
> (D)(1+(4/5)^25)/2
> (E)(1+(3/5)^25)/2
>
> Tento de todas as formar usar a distribuição binomial, alguma recorrência,
> mas sem sucesso.
> Bianca
>
> --
> Esta mensagem foi verificada pelo sistema de antivírus e
> acredita-se estar livre de perigo.
>
-- 
Esdras Muniz Mota
Mestrando em Matemática
Universidade Federal do Ceará

-- 
Esta mensagem foi verificada pelo sistema de antiv�rus e
 acredita-se estar livre de perigo.



[obm-l] Seria por distribuição binomial ou alguma recorrência

2023-02-28 Por tôpico Bianca Flores
Alguém poderia ajudar com essa questão: estou frustrada porque não consigo 
chegar ao gabarito E.

Um estudante preenche, aleatoriamente e de forma independente cada uma das 
questões, um exame de múltipla escolha com 5 respostas possíveis (das quais 
apenas uma é correta) para cada uma de 25 questões. A probabilidade que ele 
acerte um número par de questões é dada por:

(A)(1-(4/5)^25)/2
(B)(1-(3/5)^25)/2
(C)((3/5)^25)/2
(D)(1+(4/5)^25)/2
(E)(1+(3/5)^25)/2

Tento de todas as formar usar a distribuição binomial, alguma recorrência, mas 
sem sucesso.
Bianca

-- 
Esta mensagem foi verificada pelo sistema de antivírus e
 acredita-se estar livre de perigo.



[obm-l] Função uniformemente diferenciável

2023-01-26 Por tôpico Artur Costa Steiner
Alguém já ouviu falar neste conceito? Acho que quase não é
difundido.Dizemos que f:I --> R é uniformente diferenciável no intervalo I
se, para todo eps > 0, houver d > 0 tal que, se x e y estiverem em I e 0 <
|y - x| < d, então |((f(y - f(x))/(y - x)) - f'(x)| < eps. Isto significa
que, no limite definindo derivada, dado eps existe um mesmo d que é bom
para todo x de I. d depende só de eps, não de x.

Por exemplo, se I = (0, 1), f(x) = x^2 é uniformente diferenciável em I,
mas g(x) = ln(x) não é.

f é uniformente diferenciável em I se, e somente se, f' for uniformente
contínua em I.Talvez por causa desta equivalência o conceito de
diferenciabilidade uniforme seja pouco difundido.

O conceito pode ser extendido para o domínio complexo, caso em que I é
substituído por uma região S (conjunto aberto e conexo) do plano complexo.
Como no caso real, diferenciabilidade uniforme de f em S implica
continuidade uniforme de f'  em S. Mas a recíproca creio que não vale.

Mas no caso complexo há uma conclusão interessante: f é uniformemente
diferenciável em todo o plano C se, e somente se, f for um polinômio de
grau <= 2.

Abrs
Artur

-- 
Esta mensagem foi verificada pelo sistema de antiv�rus e
 acredita-se estar livre de perigo.



[obm-l] Re: [obm-l] Re: [obm-l] Re: [obm-l] cadeira de 3 pés

2023-01-23 Por tôpico Bernardo Freitas Paulo da Costa
On Mon, Jan 23, 2023 at 12:52 PM Claudio Buffara
 wrote:
>
> Obrigado, Wagner e Ponce:
>
> Eu tinha pensado em algo na linha do que o Ponce escreveu, ainda que em 
> certos casos patológicos (pelo menos de piso...) o terceiro pé pode não 
> encontrar apoio: imagine um piso com um pico fino em algum lugar (p.ex. a 
> superfície gerada pela revolução de z = 1/(1+(x^2+y^2)^20) em torno do eixo 
> z). Neste caso, precisaríamos de uma cadeira bem pequena, ou pelo menos com 
> as pontas dos pés bem próximas umas das outras.

Pois é, eu acho que tem uma questão interessante de estabilidade que é
a seguinte: se você pegar um banco de 3 pés e botar na encosta de uma
montanha (bem íngreme!), ele vai rolar ladeira abaixo.  Porque a
projeção do centro de gravidade cai fora do triângulo determinado pelo
contato com o solo (que define as normais que podem segurar a cadeira
em posição estática).  Uma cadeira de 4 pés só é bamba porque uma
pessoa não consegue ficar (infinitesimalmente) parada, e daí o centro
de gravidade oscila entre os triângulos formados por dois pares de
três apoios, por exemplo ABC e ACD.

Bernardo

-- 
Esta mensagem foi verificada pelo sistema de antiv�rus e
 acredita-se estar livre de perigo.


=
Instru��es para entrar na lista, sair da lista e usar a lista em
http://www.mat.puc-rio.br/~obmlistas/obm-l.html
=


[obm-l] Re: [obm-l] Re: [obm-l] cadeira de 3 pés

2023-01-23 Por tôpico Claudio Buffara
" No mundo 2D um banco de 2 pernas não balança, mas um de 3 pernas pode
balançar."

Aqui estamos falando de uma curva no plano e de um número real positivo d,
suficientemente pequeno de modo que, para todo ponto A na curva, existe um
ponto B na curva com dist(A,B) = d.
A e B são as pontas das pernas do banco 2D.
Isso será verdade se todo ponto da curva for o centro de uma circunferência
de raio d que intersecta a curva.

Um terceiro ponto C, a distâncias predeterminadas de A e B, pode não
pertencer à curva (ou seja, o triângulo ABC não estará inscrito na curva).
Mas é possível deslizar a corda AB, de comprimento constante, ao longo de
toda a curva.

No caso 3D, temos uma superfície no espaço e 3 números reais positivos a,
b, c, com cada um menor do que a soma dos outros dois, de modo que, para
cada A na superfície, é possível achar B e C na superfície com dist(A,B) =
c, dist(A,C) = b e dist(B,C) = a.
E é possível deslizar o triângulo ABC, com lados de comprimento constante,
ao longo de toda a superfície.
Mas, fixado A na superfície, não me parece tão simples construir B e C.


On Mon, Jan 23, 2023 at 11:29 AM Eduardo Wagner  wrote:

> O banco de 3 pernas não balança porque nosso mundo é tridimensional.
> Não tem nada a ver com plano ou triângulo. Um banco de 3 pernas não
> balança se for colocado
> no teto de um carro.
> No mundo 2D um banco de 2 pernas não balança, mas um de 3 pernas pode
> balançar.
> Em um mundo 4D uma cadeira de 4 pernas não balança.
> Wbs
> Wagner
>
>
> Em dom., 22 de jan. de 2023 às 23:24, Claudio Buffara <
> claudio.buff...@gmail.com> escreveu:
>
>> Achei na internet duas explicações distintas para a estabilidade de uma
>> cadeira (ou mesa ou banco) de 3 pés.
>> Aqui estão:
>> https://www.somatematica.com.br/curiosidades/c98.php
>>
>> http://colegiofarroupilha.com.br/site/qual-cadeira-e-mais-firme-a-que-tem-tres-ou-quatro-pes/
>>
>> Qual das duas é a explicação correta?
>> Ou nenhuma das duas? E, nesse caso, qual a explicação?
>>
>> []s,
>> Claudio.
>>
>> --
>> Esta mensagem foi verificada pelo sistema de antivírus e
>> acredita-se estar livre de perigo.
>
>
> --
> Esta mensagem foi verificada pelo sistema de antivírus e
> acredita-se estar livre de perigo.

-- 
Esta mensagem foi verificada pelo sistema de antiv�rus e
 acredita-se estar livre de perigo.



[obm-l] Re: [obm-l] Re: [obm-l] cadeira de 3 pés

2023-01-23 Por tôpico Claudio Buffara
Obrigado, Wagner e Ponce:

Eu tinha pensado em algo na linha do que o Ponce escreveu, ainda que em
certos casos patológicos (pelo menos de piso...) o terceiro pé pode não
encontrar apoio: imagine um piso com um pico fino em algum lugar (p.ex. a
superfície gerada pela revolução de z = 1/(1+(x^2+y^2)^20) em torno do eixo
z). Neste caso, precisaríamos de uma cadeira bem pequena, ou pelo menos com
as pontas dos pés bem próximas umas das outras.

Mas, pelo que o Wagner escreveu, acho que ainda tem um teorema mais
profundo aí.

[]s,
Claudio.


On Mon, Jan 23, 2023 at 11:54 AM Rogerio Ponce  wrote:

> Ola' Claudio!
> Eu diria que as duas explicações estão erradas, pois não se depende de
> ter apenas um plano definido pelas pontas dos pés, visto que uma
> cadeira de 4 pés pode, perfeitamente, ter as pontas dos 4 pés em
> apenas um plano, e, ainda assim, ela não é necessariamente estável.
>
> Um explicação menos ruim é que, numa cadeira de 3 pés, sempre podemos
> apoiar quaisquer 2 pés num piso (mesmo irregular), e, em torno do eixo
> definido pelos 2 pés já apoiados, podemos girar a cadeira até que o
> terceiro pé encontre o piso, de modo que a cadeira fique totalmente
> apoiada.
> Já numa cadeira de 4 pés, é comum que um dos pés fique sem contato com
> o chão, permitindo que a cadeira oscile em torno do eixo definido
> pelos 2 pés vizinhos ao pé sem contato.
>
> []'s
> Rogerio Ponce
>
> On Sun, Jan 22, 2023 at 11:23 PM Claudio Buffara
>  wrote:
> >
> > Achei na internet duas explicações distintas para a estabilidade de uma
> cadeira (ou mesa ou banco) de 3 pés.
> > Aqui estão:
> > https://www.somatematica.com.br/curiosidades/c98.php
> >
> http://colegiofarroupilha.com.br/site/qual-cadeira-e-mais-firme-a-que-tem-tres-ou-quatro-pes/
> >
> > Qual das duas é a explicação correta?
> > Ou nenhuma das duas? E, nesse caso, qual a explicação?
> >
> > []s,
> > Claudio.
> >
> > --
> > Esta mensagem foi verificada pelo sistema de antivírus e
> > acredita-se estar livre de perigo.
>
> --
> Esta mensagem foi verificada pelo sistema de antivírus e
>  acredita-se estar livre de perigo.
>
>
> =
> Instru�ões para entrar na lista, sair da lista e usar a lista em
> http://www.mat.puc-rio.br/~obmlistas/obm-l.html
> =
>

-- 
Esta mensagem foi verificada pelo sistema de antiv�rus e
 acredita-se estar livre de perigo.



[obm-l] Re: [obm-l] Re: [obm-l] cadeira de 3 pés

2023-01-23 Por tôpico Anderson Torres
Em seg, 23 de jan de 2023 11:54, Rogerio Ponce 
escreveu:

> Ola' Claudio!
> Eu diria que as duas explicações estão erradas, pois não se depende de
> ter apenas um plano definido pelas pontas dos pés, visto que uma
> cadeira de 4 pés pode, perfeitamente, ter as pontas dos 4 pés em
> apenas um plano, e, ainda assim, ela não é necessariamente estável.
>

Como não? A ideia de botar um calço é precisamente estabilizar o pé que não
encosta na superfície. E, exceto em feiras de ciências e circos, o chão
costuma ser plano.


> Um explicação menos ruim é que, numa cadeira de 3 pés, sempre podemos
> apoiar quaisquer 2 pés num piso (mesmo irregular), e, em torno do eixo
> definido pelos 2 pés já apoiados, podemos girar a cadeira até que o
> terceiro pé encontre o piso, de modo que a cadeira fique totalmente
> apoiada.
> Já numa cadeira de 4 pés, é comum que um dos pés fique sem contato com
> o chão, permitindo que a cadeira oscile em torno do eixo definido
> pelos 2 pés vizinhos ao pé sem contato.
>
> []'s
> Rogerio Ponce
>
> On Sun, Jan 22, 2023 at 11:23 PM Claudio Buffara
>  wrote:
> >
> > Achei na internet duas explicações distintas para a estabilidade de uma
> cadeira (ou mesa ou banco) de 3 pés.
> > Aqui estão:
> > https://www.somatematica.com.br/curiosidades/c98.php
> >
> http://colegiofarroupilha.com.br/site/qual-cadeira-e-mais-firme-a-que-tem-tres-ou-quatro-pes/
> >
> > Qual das duas é a explicação correta?
> > Ou nenhuma das duas? E, nesse caso, qual a explicação?
> >
> > []s,
> > Claudio.
> >
> > --
> > Esta mensagem foi verificada pelo sistema de antivírus e
> > acredita-se estar livre de perigo.
>
> --
> Esta mensagem foi verificada pelo sistema de antivírus e
>  acredita-se estar livre de perigo.
>
>
> =
> Instru�ões para entrar na lista, sair da lista e usar a lista em
> http://www.mat.puc-rio.br/~obmlistas/obm-l.html
> =
>

-- 
Esta mensagem foi verificada pelo sistema de antiv�rus e
 acredita-se estar livre de perigo.



[obm-l] Re: [obm-l] Re: [obm-l] Re: [obm-l] cadeira de 3 pés

2023-01-23 Por tôpico Anderson Torres
Em seg, 23 de jan de 2023 11:15, Claudio Buffara 
escreveu:

> Será que o argumento usando apenas o plano é suficiente?  Pois um banco de
> 3 pés também fica estável num piso irregular.
>

Mas nem toda cadeira de quatro pés fica estável em qualquer piso irregular.

A ideia subjacente ainda é a de "planidade". Os pés da cadeira estão na
intersecção entre o "plano dos pés" e o piso.
Mas o mesmo não se aplica às cadeiras quadrúpedes, pois nem sempre existe
um "plano dos pés".

Se bem que neste último caso é possível que os pés da mesa sejam coplanares
mas a superfície não o seja. Ou pior ainda, forçando um pouco na
continuidade, pode ser que qualquer cadeira de quatro pés se encaixe em
qualquer superfície não-plana.

Ou não. Se pegarmos por exemplo o mapa topográfico de uma planície com um
poço escavado e a distância entre dois pés da mesa sempre for maior que o
diâmetro do poço, não tem como encaixar uma mesa torta de quatro pés.

Estranho...

O resultado mais geral em que pensei foi o seguinte: dada qualquer
> superfície bi-dimensional contínua (por exemplo, que seja o gráfico de uma
> função contínua de RxR em R - uma suposição razoável se estamos tentando
> modelar um piso), você sempre consegue encostar nela as pontas dos 3 pés do
> banco, de modo que o banco fique "firme" ou sem folgas.
> Ou, mais formalmente, dado um triângulo ABC no espaço, existe uma
> isometria (do espaço) tal que as imagens de A, B e C por esta isometria
> estão em S.
>

Pensei algo do gênero, mas de maneira bem menos elaborada: dados um
triângulo ABC e dois pontos A',B' em uma superfície contínua que distam AB,
existe um ponto C´ tal que ABC = A'B'C'


> Enfim, esse talvez seja um problema mais de topologia do que de geometria.
> Pois, no fim das contas, "3 pontos não colineares determinam um único
> plano", assim como "2 pontos determinam uma única reta" são afirmações que
> têm um certo ar topológico, pelo menos pra mim.
>

> []s,
> Claudio.
>
> On Mon, Jan 23, 2023 at 7:02 AM Anderson Torres <
> torres.anderson...@gmail.com> wrote:
>
>>
>>
>> Em dom, 22 de jan de 2023 23:23, Claudio Buffara <
>> claudio.buff...@gmail.com> escreveu:
>>
>>> Achei na internet duas explicações distintas para a estabilidade de uma
>>> cadeira (ou mesa ou banco) de 3 pés.
>>> Aqui estão:
>>> https://www.somatematica.com.br/curiosidades/c98.php
>>>
>>> http://colegiofarroupilha.com.br/site/qual-cadeira-e-mais-firme-a-que-tem-tres-ou-quatro-pes/
>>>
>>
>> Nesse caso específico, a primeira me parece mais correta. Ou melhor, a
>> segunda tem falhas.
>>
>> A rigidez dos triângulos (TCC caso LLL de igualdade de triângulos) é
>> irrelevante para a questão da cadeira bamba. Em qualquer cadeira física e
>> palpável, as pernas são rígidas - portanto o polígono formado pelas
>> extremidades dessas pernas é rígido também.
>>
>> Mas um polígono rígido não é necessariamente um polígono bidimensional -
>> o que a cadeira bamba de 4 pernas exemplifica perfeitamente.
>>
>> Contate o webmaster da segunda página sugerindo correções!
>>
>>
>>> Qual das duas é a explicação correta?
>>> Ou nenhuma das duas? E, nesse caso, qual a explicação?
>>>
>>> []s,
>>> Claudio.
>>>
>>> --
>>> Esta mensagem foi verificada pelo sistema de antivírus e
>>> acredita-se estar livre de perigo.
>>
>>
>> --
>> Esta mensagem foi verificada pelo sistema de antivírus e
>> acredita-se estar livre de perigo.
>
>
> --
> Esta mensagem foi verificada pelo sistema de antivírus e
> acredita-se estar livre de perigo.

-- 
Esta mensagem foi verificada pelo sistema de antiv�rus e
 acredita-se estar livre de perigo.



[obm-l] Re: [obm-l] cadeira de 3 pés

2023-01-23 Por tôpico Rogerio Ponce
Ola' Claudio!
Eu diria que as duas explicações estão erradas, pois não se depende de
ter apenas um plano definido pelas pontas dos pés, visto que uma
cadeira de 4 pés pode, perfeitamente, ter as pontas dos 4 pés em
apenas um plano, e, ainda assim, ela não é necessariamente estável.

Um explicação menos ruim é que, numa cadeira de 3 pés, sempre podemos
apoiar quaisquer 2 pés num piso (mesmo irregular), e, em torno do eixo
definido pelos 2 pés já apoiados, podemos girar a cadeira até que o
terceiro pé encontre o piso, de modo que a cadeira fique totalmente
apoiada.
Já numa cadeira de 4 pés, é comum que um dos pés fique sem contato com
o chão, permitindo que a cadeira oscile em torno do eixo definido
pelos 2 pés vizinhos ao pé sem contato.

[]'s
Rogerio Ponce

On Sun, Jan 22, 2023 at 11:23 PM Claudio Buffara
 wrote:
>
> Achei na internet duas explicações distintas para a estabilidade de uma 
> cadeira (ou mesa ou banco) de 3 pés.
> Aqui estão:
> https://www.somatematica.com.br/curiosidades/c98.php
> http://colegiofarroupilha.com.br/site/qual-cadeira-e-mais-firme-a-que-tem-tres-ou-quatro-pes/
>
> Qual das duas é a explicação correta?
> Ou nenhuma das duas? E, nesse caso, qual a explicação?
>
> []s,
> Claudio.
>
> --
> Esta mensagem foi verificada pelo sistema de antivírus e
> acredita-se estar livre de perigo.

-- 
Esta mensagem foi verificada pelo sistema de antiv�rus e
 acredita-se estar livre de perigo.


=
Instru��es para entrar na lista, sair da lista e usar a lista em
http://www.mat.puc-rio.br/~obmlistas/obm-l.html
=


[obm-l] Re: [obm-l] cadeira de 3 pés

2023-01-23 Por tôpico Eduardo Wagner
O banco de 3 pernas não balança porque nosso mundo é tridimensional.
Não tem nada a ver com plano ou triângulo. Um banco de 3 pernas não balança
se for colocado
no teto de um carro.
No mundo 2D um banco de 2 pernas não balança, mas um de 3 pernas pode
balançar.
Em um mundo 4D uma cadeira de 4 pernas não balança.
Wbs
Wagner


Em dom., 22 de jan. de 2023 às 23:24, Claudio Buffara <
claudio.buff...@gmail.com> escreveu:

> Achei na internet duas explicações distintas para a estabilidade de uma
> cadeira (ou mesa ou banco) de 3 pés.
> Aqui estão:
> https://www.somatematica.com.br/curiosidades/c98.php
>
> http://colegiofarroupilha.com.br/site/qual-cadeira-e-mais-firme-a-que-tem-tres-ou-quatro-pes/
>
> Qual das duas é a explicação correta?
> Ou nenhuma das duas? E, nesse caso, qual a explicação?
>
> []s,
> Claudio.
>
> --
> Esta mensagem foi verificada pelo sistema de antivírus e
> acredita-se estar livre de perigo.

-- 
Esta mensagem foi verificada pelo sistema de antiv�rus e
 acredita-se estar livre de perigo.



[obm-l] Re: [obm-l] Re: [obm-l] cadeira de 3 pés

2023-01-23 Por tôpico Claudio Buffara
Será que o argumento usando apenas o plano é suficiente?  Pois um banco de
3 pés também fica estável num piso irregular.
O resultado mais geral em que pensei foi o seguinte: dada qualquer
superfície bi-dimensional contínua (por exemplo, que seja o gráfico de uma
função contínua de RxR em R - uma suposição razoável se estamos tentando
modelar um piso), você sempre consegue encostar nela as pontas dos 3 pés do
banco, de modo que o banco fique "firme" ou sem folgas.
Ou, mais formalmente, dado um triângulo ABC no espaço, existe uma isometria
(do espaço) tal que as imagens de A, B e C por esta isometria estão em S.

Enfim, esse talvez seja um problema mais de topologia do que de geometria.
Pois, no fim das contas, "3 pontos não colineares determinam um único
plano", assim como "2 pontos determinam uma única reta" são afirmações que
têm um certo ar topológico, pelo menos pra mim.

[]s,
Claudio.

On Mon, Jan 23, 2023 at 7:02 AM Anderson Torres <
torres.anderson...@gmail.com> wrote:

>
>
> Em dom, 22 de jan de 2023 23:23, Claudio Buffara <
> claudio.buff...@gmail.com> escreveu:
>
>> Achei na internet duas explicações distintas para a estabilidade de uma
>> cadeira (ou mesa ou banco) de 3 pés.
>> Aqui estão:
>> https://www.somatematica.com.br/curiosidades/c98.php
>>
>> http://colegiofarroupilha.com.br/site/qual-cadeira-e-mais-firme-a-que-tem-tres-ou-quatro-pes/
>>
>
> Nesse caso específico, a primeira me parece mais correta. Ou melhor, a
> segunda tem falhas.
>
> A rigidez dos triângulos (TCC caso LLL de igualdade de triângulos) é
> irrelevante para a questão da cadeira bamba. Em qualquer cadeira física e
> palpável, as pernas são rígidas - portanto o polígono formado pelas
> extremidades dessas pernas é rígido também.
>
> Mas um polígono rígido não é necessariamente um polígono bidimensional - o
> que a cadeira bamba de 4 pernas exemplifica perfeitamente.
>
> Contate o webmaster da segunda página sugerindo correções!
>
>
>> Qual das duas é a explicação correta?
>> Ou nenhuma das duas? E, nesse caso, qual a explicação?
>>
>> []s,
>> Claudio.
>>
>> --
>> Esta mensagem foi verificada pelo sistema de antivírus e
>> acredita-se estar livre de perigo.
>
>
> --
> Esta mensagem foi verificada pelo sistema de antivírus e
> acredita-se estar livre de perigo.

-- 
Esta mensagem foi verificada pelo sistema de antiv�rus e
 acredita-se estar livre de perigo.



[obm-l] Re: [obm-l] cadeira de 3 pés

2023-01-23 Por tôpico Anderson Torres
Em dom, 22 de jan de 2023 23:23, Claudio Buffara 
escreveu:

> Achei na internet duas explicações distintas para a estabilidade de uma
> cadeira (ou mesa ou banco) de 3 pés.
> Aqui estão:
> https://www.somatematica.com.br/curiosidades/c98.php
>
> http://colegiofarroupilha.com.br/site/qual-cadeira-e-mais-firme-a-que-tem-tres-ou-quatro-pes/
>

Nesse caso específico, a primeira me parece mais correta. Ou melhor, a
segunda tem falhas.

A rigidez dos triângulos (TCC caso LLL de igualdade de triângulos) é
irrelevante para a questão da cadeira bamba. Em qualquer cadeira física e
palpável, as pernas são rígidas - portanto o polígono formado pelas
extremidades dessas pernas é rígido também.

Mas um polígono rígido não é necessariamente um polígono bidimensional - o
que a cadeira bamba de 4 pernas exemplifica perfeitamente.

Contate o webmaster da segunda página sugerindo correções!


> Qual das duas é a explicação correta?
> Ou nenhuma das duas? E, nesse caso, qual a explicação?
>
> []s,
> Claudio.
>
> --
> Esta mensagem foi verificada pelo sistema de antivírus e
> acredita-se estar livre de perigo.

-- 
Esta mensagem foi verificada pelo sistema de antiv�rus e
 acredita-se estar livre de perigo.



[obm-l] cadeira de 3 pés

2023-01-22 Por tôpico Claudio Buffara
Achei na internet duas explicações distintas para a estabilidade de uma
cadeira (ou mesa ou banco) de 3 pés.
Aqui estão:
https://www.somatematica.com.br/curiosidades/c98.php
http://colegiofarroupilha.com.br/site/qual-cadeira-e-mais-firme-a-que-tem-tres-ou-quatro-pes/

Qual das duas é a explicação correta?
Ou nenhuma das duas? E, nesse caso, qual a explicação?

[]s,
Claudio.

-- 
Esta mensagem foi verificada pelo sistema de antiv�rus e
 acredita-se estar livre de perigo.



Re: [obm-l] Irracionalidade de Pi

2023-01-22 Por tôpico Claudio Buffara
Valeu!
E os links extras contém uma boa discussão, às vezes meio acalorada, sobre
motivação pra certas demonstrações.
Eu particularmente me interesso bastante por este tema.
Pois acho que demonstrações "mágicas", baseadas em ideias "vindas do além",
são problemáticas do ponto de vista pedagógico, pois acho que podem
desmotivar estudantes de matemática, que passam a achar que o assunto é só
pra gênios.
Estas não devem ser confundidas com demonstrações/soluções brilhantes mas
que são "óbvias a posteriori", ou seja, que dependem de uma sacada que o
leitor poderia ter tido se tivesse prestado mais atenção ou feito um
desenho mais preciso ou pensado um pouquinho mais no problema.  Não me
parece ser o caso dessa demonstração do Niven da irracionalidade de Pi.

[]s,
Claudio.




On Sun, Jan 22, 2023 at 9:53 AM Anderson Torres <
torres.anderson...@gmail.com> wrote:

> Em sáb., 21 de jan. de 2023 às 13:27, Claudio Buffara
>  escreveu:
> >
> > A demonstração tradicional da irracionalidade de Pi começa estabelecendo
> algumas propriedades da função:
> > x |--> x^n * (1-x)^n / n!
> > no intervalo (0,1).
> >
> > Essa função me parece tirada da cartola, sem qualquer motivação prévia.
> > Alguém sabe o que levou o autor da demonstração a usar esta função?
>
> Bem, eu fiz uma rápida busca no Google por "motivated demonstration
> irrationality pi" e encontrei isso:
>
> "Discovering and Proving that π Is Irrational" por Timothy W. Jones
>
> The American Mathematical Monthly
> Vol. 117, No. 6 (June-July 2010), pp. 553-557 (5 pages)
> https://doi.org/10.4169/000298910x492853
>
> E também uns links extras:
>
>
> https://mattbaker.blog/2015/03/15/a-motivated-and-simple-proof-that-pi-is-irrational/
>
> https://math.stackexchange.com/questions/4051354/what-is-the-motivation-behind-the-steps-in-this-simple-proof-that-pi-is-irr
> https://page.math.tu-berlin.de/~mdmv/archive/19/mdmv-19-zhou.pdf
>
> Divirta-se :)
>
> >
> > []s,
> > Claudio.
> >
> > --
> > Esta mensagem foi verificada pelo sistema de antivírus e
> > acredita-se estar livre de perigo.
>
> --
> Esta mensagem foi verificada pelo sistema de antivírus e
>  acredita-se estar livre de perigo.
>
>
> =
> Instru�ões para entrar na lista, sair da lista e usar a lista em
> http://www.mat.puc-rio.br/~obmlistas/obm-l.html
> =
>

-- 
Esta mensagem foi verificada pelo sistema de antiv�rus e
 acredita-se estar livre de perigo.



Re: [obm-l] Irracionalidade de Pi

2023-01-22 Por tôpico Anderson Torres
Em sáb., 21 de jan. de 2023 às 13:27, Claudio Buffara
 escreveu:
>
> A demonstração tradicional da irracionalidade de Pi começa estabelecendo 
> algumas propriedades da função:
> x |--> x^n * (1-x)^n / n!
> no intervalo (0,1).
>
> Essa função me parece tirada da cartola, sem qualquer motivação prévia.
> Alguém sabe o que levou o autor da demonstração a usar esta função?

Bem, eu fiz uma rápida busca no Google por "motivated demonstration
irrationality pi" e encontrei isso:

"Discovering and Proving that π Is Irrational" por Timothy W. Jones

The American Mathematical Monthly
Vol. 117, No. 6 (June-July 2010), pp. 553-557 (5 pages)
https://doi.org/10.4169/000298910x492853

E também uns links extras:

https://mattbaker.blog/2015/03/15/a-motivated-and-simple-proof-that-pi-is-irrational/
https://math.stackexchange.com/questions/4051354/what-is-the-motivation-behind-the-steps-in-this-simple-proof-that-pi-is-irr
https://page.math.tu-berlin.de/~mdmv/archive/19/mdmv-19-zhou.pdf

Divirta-se :)

>
> []s,
> Claudio.
>
> --
> Esta mensagem foi verificada pelo sistema de antivírus e
> acredita-se estar livre de perigo.

-- 
Esta mensagem foi verificada pelo sistema de antiv�rus e
 acredita-se estar livre de perigo.


=
Instru��es para entrar na lista, sair da lista e usar a lista em
http://www.mat.puc-rio.br/~obmlistas/obm-l.html
=


[obm-l] Irracionalidade de Pi

2023-01-21 Por tôpico Claudio Buffara
A demonstração tradicional da irracionalidade de Pi começa estabelecendo
algumas propriedades da função:
x |--> x^n * (1-x)^n / n!
no intervalo (0,1).

Essa função me parece tirada da cartola, sem qualquer motivação prévia.
Alguém sabe o que levou o autor da demonstração a usar esta função?

[]s,
Claudio.

-- 
Esta mensagem foi verificada pelo sistema de antiv�rus e
 acredita-se estar livre de perigo.



[obm-l] Re: [obm-l] Mostrar que [n!]/e é sempre par

2022-12-15 Por tôpico Anderson Torres
Em sex, 16 de dez de 2022 00:53, Artur Costa Steiner <
artur.costa.stei...@gmail.com> escreveu:

> Problema interessante: Mostre que, para todo inteiro n >= 0,  [n!]/e é
> sempre par, sendo [x] o piso de x.
>

você quis dizer [n!/e] onde e é a base do log natural?

Bem, 1/e=e^(-1)=
(1/0!-1/1!)+(1/2!-1/3!)+(1/4!-1/5!)+(1/6!-1/7!)+... =
2/3! + 4/5! + 6/7! + 8/9! + ...

Dessa forma, [n!/e]=

[2n!/3! + 4n!/5! + 6n!/7! + 8n!/9! + ...]

Que, obviamente, redunda numa sema de pares após truncado (denominador
>n!),


> Abraços
>
> Artur
>
> --
> Esta mensagem foi verificada pelo sistema de antivírus e
> acredita-se estar livre de perigo.

-- 
Esta mensagem foi verificada pelo sistema de antiv�rus e
 acredita-se estar livre de perigo.



[obm-l] Mostrar que [n!]/e é sempre par

2022-12-15 Por tôpico Artur Costa Steiner
Problema interessante: Mostre que, para todo inteiro n >= 0,  [n!]/e é
sempre par, sendo [x] o piso de x.

Abraços

Artur

-- 
Esta mensagem foi verificada pelo sistema de antiv�rus e
 acredita-se estar livre de perigo.



[obm-l] Probabilidade

2022-12-15 Por tôpico Claudio Buffara
Alguém conhece alguma boa referência para métodos de resolução de problemas
do tipo:
1) Joga-se uma moeda 1000 vezes. Qual a probabilidade de se ter uma
sequência de exatamente 20 caras consecutivas?  De pelo menos 20 caras
consecutivas?
1a) Analogamente com um dado.
2) Dadas 100 amostras independentes de uma distribuição uniforme em [0,1],
qual a probabilidade de (exatamente ou pelo menos) 20 delas estarem no
intervalo [2/5,1/2]?  Qual a probabilidade de 20 delas estarem num
intervalo de comprimento 1/10?

Também tenho interesse em bibliografia sobre testes de aleatoriedade
aplicados a situações como as acima (p.ex. dados os resultados de 1000
lançamentos de uma moeda, qual a probabilidade da moeda ser honesta/dos
lançamentos terem sido independentes)

[]s,
Claudio.

-- 
Esta mensagem foi verificada pelo sistema de antiv�rus e
 acredita-se estar livre de perigo.



[obm-l] Re: [obm-l] Teoria dos números, trigonometria e racionalidade

2022-12-11 Por tôpico Anderson Torres
Em dom., 11 de dez. de 2022 às 10:32, Anderson Torres
 escreveu:
>
> Em sáb., 10 de dez. de 2022 às 22:08, marcone augusto araújo borges
>  escreveu:
> >
> > Seja p um número primo tal que p = = 3 (mod4) e @ um ângulo tal que tan@ é 
> > racional. Prove que tan((p+1)@) também é racional com numerador múltiplo de 
> > p
> > Desde já agradeço por algum esclarecimento ou solução.
>
> Bem, o que eu consigo pensar é em algo desse tipo.
>
> Sabemos que tan(m+n) = (tan(m) + tan(n))/(1-tan(m)* tan(n))
>
> Escrevamos tan(nX)=p(n)/q(n), onde p e q são polinômios em t=tan(X).
> Temos então a seguinte recorrência:
>
> p(1)=t; p(n+1)=p(n)+tq(n)
> q(1)=1; q(n+1)=-tp(n)+q(n)
>
> Jogando aqui e ali, temos
>
> p(1)=t; p(2)=2t; p(n+2)=2p(n+1)-(t^2+1)p(n)
> q(1)=1; q(2)=1-t^2; q(n+2)=2q(n+1)-(t^2+1)q(n)
>
> De cara, se nota que p sempre será múltiplo de p, e que q sempre deixa
> resto 1 módulo t, o que já dá uma pista do que procurar...
> Decerto, vai aparecer alguma coisa do tipo x^2+1, e com isso se usa o
> fato de p ser primo da forma 4k-1...
>

Acho que dá para melhorar. Suponha tan(nX)=A(n)/B(n). Assim,

A(n+1) =  B*A(n) + A*B(n)
B(n+1) = -A*A(n) + B*B(n)

E portanto

A(n+2) = 2B*A(n+1) - (A^2+B^2)*A(n), A(1)=A, A(2)=2AB
B(n+2) = 2B*B(n+1) - (A^2+B^2)*B(n), B(1)=B, B(2)=B^2-A^2

A ideia então seria demonstrar que A(p+1) é múltiplo de p para p primo
da forma 4k-1, e B(p+1) não é múltiplo de p para p primo da forma
4k-1.

Dessa forma, ao menos em princípio seria possível verificar a segunda
premissa, pois a primeira é óbvia.

> >
> > --
> > Esta mensagem foi verificada pelo sistema de antivírus e
> > acredita-se estar livre de perigo.

-- 
Esta mensagem foi verificada pelo sistema de antiv�rus e
 acredita-se estar livre de perigo.


=
Instru��es para entrar na lista, sair da lista e usar a lista em
http://www.mat.puc-rio.br/~obmlistas/obm-l.html
=


[obm-l] Re: [obm-l] OBM e Olímpiadas internacionais

2022-12-11 Por tôpico Anderson Torres
Em qua., 7 de dez. de 2022 às 03:39, Obindinachukwu Desire Yema
 escreveu:
>
> Bom dia a todos,
> Nesse ano eu despertei um interesse em matemática pura, pensando um pouco 
> decidi que iria tentar no próximo ano fazer a OBM nivel universitário. 
> Pesquisando no site da OBM, eu não achei nada relacionado com o conteúdo que 
> cai na prova.
> Eu queria perguntar para vocês como que me preparo para a prova, no sentido 
> de: conteúdo que devo saber.
> Desde já agradeço a atenção.

De fato tem pouca coisa além das provas. Então, te sugiro pegar
pesado: estude a PUTNAM e a IMC. Com isso você vai ter mais material.

>
> --
> Esta mensagem foi verificada pelo sistema de antivírus e
> acredita-se estar livre de perigo.

-- 
Esta mensagem foi verificada pelo sistema de antiv�rus e
 acredita-se estar livre de perigo.


=
Instru��es para entrar na lista, sair da lista e usar a lista em
http://www.mat.puc-rio.br/~obmlistas/obm-l.html
=


[obm-l] Re: [obm-l] Teoria dos números, trigonometria e racionalidade

2022-12-11 Por tôpico Anderson Torres
Em sáb., 10 de dez. de 2022 às 22:08, marcone augusto araújo borges
 escreveu:
>
> Seja p um número primo tal que p = = 3 (mod4) e @ um ângulo tal que tan@ é 
> racional. Prove que tan((p+1)@) também é racional com numerador múltiplo de p
> Desde já agradeço por algum esclarecimento ou solução.

Bem, o que eu consigo pensar é em algo desse tipo.

Sabemos que tan(m+n) = (tan(m) + tan(n))/(1-tan(m)* tan(n))

Escrevamos tan(nX)=p(n)/q(n), onde p e q são polinômios em t=tan(X).
Temos então a seguinte recorrência:

p(1)=t; p(n+1)=p(n)+tq(n)
q(1)=1; q(n+1)=-tp(n)+q(n)

Jogando aqui e ali, temos

p(1)=t; p(2)=2t; p(n+2)=2p(n+1)-(t^2+1)p(n)
q(1)=1; q(2)=1-t^2; q(n+2)=2q(n+1)-(t^2+1)q(n)

De cara, se nota que p sempre será múltiplo de p, e que q sempre deixa
resto 1 módulo t, o que já dá uma pista do que procurar...
Decerto, vai aparecer alguma coisa do tipo x^2+1, e com isso se usa o
fato de p ser primo da forma 4k-1...

>
> --
> Esta mensagem foi verificada pelo sistema de antivírus e
> acredita-se estar livre de perigo.

-- 
Esta mensagem foi verificada pelo sistema de antiv�rus e
 acredita-se estar livre de perigo.


=
Instru��es para entrar na lista, sair da lista e usar a lista em
http://www.mat.puc-rio.br/~obmlistas/obm-l.html
=


[obm-l] Re: Teoria dos números

2022-12-11 Por tôpico marcone augusto araújo borges
Correção: não é (@+1)p, é (p+1)@


De: owner-ob...@mat.puc-rio.br  em nome de marcone 
augusto araújo borges 
Enviado: sábado, 10 de dezembro de 2022 07:38
Para: obm-l@mat.puc-rio.br 
Assunto: [obm-l] Teoria dos números

Seja p = = 3(mod4) um número primo e @ um ângulo tal que tan@ é racional. Prove 
que tan((@+1)p) é também racional com numerador múltiplo de p.
Se alguém puder resolver eu agradeço

--
Esta mensagem foi verificada pelo sistema de antivírus e
acredita-se estar livre de perigo.

-- 
Esta mensagem foi verificada pelo sistema de antivírus e
 acredita-se estar livre de perigo.



[obm-l] Teoria dos números, trigonometria e racionalidade

2022-12-10 Por tôpico marcone augusto araújo borges
Seja p um número primo tal que p = = 3 (mod4) e @ um ângulo tal que tan@ é 
racional. Prove que tan((p+1)@) também é racional com numerador múltiplo de p
Desde já agradeço por algum esclarecimento ou solução.

-- 
Esta mensagem foi verificada pelo sistema de antivírus e
 acredita-se estar livre de perigo.



[obm-l] Teoria dos números

2022-12-09 Por tôpico marcone augusto araújo borges
Seja p = = 3(mod4) um número primo e @ um ângulo tal que tan@ é racional. Prove 
que tan((@+1)p) é também racional com numerador múltiplo de p.
Se alguém puder resolver eu agradeço

-- 
Esta mensagem foi verificada pelo sistema de antivírus e
 acredita-se estar livre de perigo.



[obm-l] Re: [obm-l] OBM e Olímpiadas internacionais

2022-12-07 Por tôpico Claudio Buffara
Eu começaria olhando as provas de anos anteriores, por exemplo aqui:
https://www.obm.org.br/como-se-preparar/provas-e-gabaritos/

On Wed, Dec 7, 2022 at 3:39 AM Obindinachukwu Desire Yema <
obindinachukwu.y...@usp.br> wrote:

> Bom dia a todos,
> Nesse ano eu despertei um interesse em matemática pura, pensando um pouco
> decidi que iria tentar no próximo ano fazer a OBM nivel universitário.
> Pesquisando no site da OBM, eu não achei nada relacionado com o conteúdo
> que cai na prova.
> Eu queria perguntar para vocês como que me preparo para a prova, no
> sentido de: conteúdo que devo saber.
> Desde já agradeço a atenção.
>
> --
> Esta mensagem foi verificada pelo sistema de antivírus e
> acredita-se estar livre de perigo.

-- 
Esta mensagem foi verificada pelo sistema de antiv�rus e
 acredita-se estar livre de perigo.



[obm-l] OBM e Olímpiadas internacionais

2022-12-06 Por tôpico Obindinachukwu Desire Yema
Bom dia a todos,
Nesse ano eu despertei um interesse em matemática pura, pensando um pouco
decidi que iria tentar no próximo ano fazer a OBM nivel universitário.
Pesquisando no site da OBM, eu não achei nada relacionado com o conteúdo
que cai na prova.
Eu queria perguntar para vocês como que me preparo para a prova, no sentido de:
conteúdo que devo saber.
Desde já agradeço a atenção.

-- 
Esta mensagem foi verificada pelo sistema de antiv�rus e
 acredita-se estar livre de perigo.



[obm-l] Conjuntos

2022-11-18 Por tôpico Prof. Douglas Oliveira
Olá amigos mestres, podem me indicar bons livros de conjuntos, que trabalham
com álgebra dos conjuntos de todas as formas possíveis, por exemplo:
Trabalham com desigualdade de Bon Ferroni, mapas de Karnaugh,
relações com 4 conjuntos e etc.

Att
Prof Douglas Oliveira

-- 
Esta mensagem foi verificada pelo sistema de antiv�rus e
 acredita-se estar livre de perigo.



[obm-l] Re: [obm-l] Re: [obm-l] Re: [obm-l] Caracterização de Inteiros

2022-11-17 Por tôpico Claudio Buffara
Só completando...

Apesar de números irracionais serem conhecidos desde a época de Pitágoras
(vide a famosa historinha do pitagórico Hipaso, que supostamente foi
afogado por ter "vazado" o segredo da existência dos irracionais), me
parece que eles só começaram a realmente fazer falta no século 19, quando
os matemáticos estavam preocupados em formalizar a análise. Foi naquele
ponto que o Dedekind se deu conta da importância da completeza (essa
palavra tá certa?...) dos reais e da necessidade dela ser garantida por um
axioma. Se não me engano, o teorema que ele queria provar é o de que uma
sequência monótona e limitada de números reais sempre converge. E descobriu
que não conseguiria sem fazer uma hipótese adicional sobre o corpo
dos reais, capaz de diferenciá-lo dos racionais (também um corpo ordenado),
mas dentro do qual nem toda sequência converge (por exemplo, x(n+1) =
(1/2)*(x(n) + 2/x(n)), com x(0) = 1, a qual converge pra raiz(2): um
irracional). A princípio, bastaria ele incluir o axioma de completeza dos
reais e provar que este axioma não contradiz os demais axiomas. Mas ele foi
mais longe e acabou inventando uma construção dos reais a partir dos
racionais (via os chamados "cortes de Dedekind"). Talvez (estou
conjecturando aqui) ele só tenha feito isso por uma necessidade psicológica
de provar que algo é possível (no caso a extensão dos racionais aos reais)
exibindo uma construção explícita. Isso talvez seja um reflexo da ojeriza
que matemáticos da época tinham (e alguns poucos têm até hoje) de
argumentos não construtivos pra provar a existência de fatos ou objetos
matemáticos. Por exemplo, demonstrações usando o princípio das casas de
pombos são desse tipo.

Sobre "entender sem compreender", lembrei de um ditado do John von
Neumann: "Em matemática, você nunca entende as coisas. Apenas se acostuma
com elas."

[]s,
Claudio.

On Wed, Nov 16, 2022 at 6:52 PM Claudio Buffara 
wrote:

> Não entendi como uma homotetia poderia reduzir um par ordenado a um único
> número... enfim...
>
> O que se faz, no caso da relação de equivalência que descrevi, é
> representar o par (a,b) pela notação a-b.
> Daí, (a,b) e (c,d) são equivalentes sss a-b = c-d.
> E a novidade são os números negativos: as classes de equivalência de pares
> (a,b) com a < b, representadas, por exemplo, pelo par (0,c), onde c = b-a.
> Ou, na notação usual, -c.
>
> Mas não acho que se deva perder muito tempo com a construção de sistemas
> numéricos via classes de equivalência, estendendo naturais para inteiros
> para racionais para reais e para complexos.  Até porque é extremamente
> sacal, a cada etapa, checar que as operações usuais (+ e *), quando
> aplicadas aos novos números, têm todas as propriedades que conhecemos da
> escola.
> Essas construções foram a maneira que os matemáticos acharam pra
> formalizar os sistemas numéricos, a partir de conceitos mais básicos (no
> caso, pares ordenados e relações de equivalência) - é o programa do Hilbert
> (ou de Russell e Whitehead), de reduzir toda a matemática à teoria dos
> conjuntos.
> Mas, no fundo, esta é uma construção artificial, ex post.  Pois
> matemáticos já usavam todos os números muito antes dessa formalização ser
> inventada.
> E não acho que ela renda muitos frutos, nem pedagógicos (a menos que seu
> objetivo seja "entender sem compreender") e nem pra ampliação da fronteira
> do conhecimento, exceto colocar os sistemas numéricos numa base axiomática
> sólida.
> Em particular, no que diz respeito aos números reais, a única coisa que
> interessa é que eles são um corpo ordenado completo. Tanto é que vários
> livros de análise partem deste axioma e não se preocupam em construir os
> reais a partir dos naturais.
>
> []s,
> Claudio.
>
>
>
> On Tue, Nov 15, 2022 at 5:07 PM Pedro José  wrote:
>
>> Obrigado a você e ao Cláudio. Mas não sou criativo para inventar. Mas já
>> vi que terei que fazer uma homotetia, para as classes de equivalência para
>> representar só como um número e não como um par, creio eu.
>>
>> Cordialmente,
>> PJMS
>>
>> Em ter., 15 de nov. de 2022 às 16:00, Anderson Torres <
>> torres.anderson...@gmail.com> escreveu:
>>
>>>
>>>
>>> Em ter, 15 de nov de 2022 14:33, Pedro José 
>>> escreveu:
>>>
 Boa tarde!
 Para os |Naturais, temos os postulados de Peano.

 Para os Inteiros há alguma formalização?

>>>
>>> invente uma!
>>>
>>> Pode ser por exemplo o conjunto de pares (p,q) tais que p-q é constante.
>>>
>>> ou melhor (p1,q1)=(p2,q2) se e só se p1+q2=p2+q1.
>>>
>>>
 Acho pobre dizer que é necessário ter outros números devido ao problema
 de fechamento nos naturais para a subtração que é fato e daí introduzir os
 simétricos que são inteiros e ainda não foram caracterizados.

 No meu antigo ginásio aprendi que os Reais era a união dos conjuntos
 disjuntos irracionais e racionais. Os racionais haviam sido bem definidos.
 Aí questionei e o que são irracionais? resposta: são os Reais que não são
 racionais, os que não podem 

[obm-l] Re: [obm-l] Re: [obm-l] Re: [obm-l] Caracterização de Inteiros

2022-11-16 Por tôpico Claudio Buffara
Não entendi como uma homotetia poderia reduzir um par ordenado a um único
número... enfim...

O que se faz, no caso da relação de equivalência que descrevi, é
representar o par (a,b) pela notação a-b.
Daí, (a,b) e (c,d) são equivalentes sss a-b = c-d.
E a novidade são os números negativos: as classes de equivalência de pares
(a,b) com a < b, representadas, por exemplo, pelo par (0,c), onde c = b-a.
Ou, na notação usual, -c.

Mas não acho que se deva perder muito tempo com a construção de sistemas
numéricos via classes de equivalência, estendendo naturais para inteiros
para racionais para reais e para complexos.  Até porque é extremamente
sacal, a cada etapa, checar que as operações usuais (+ e *), quando
aplicadas aos novos números, têm todas as propriedades que conhecemos da
escola.
Essas construções foram a maneira que os matemáticos acharam pra formalizar
os sistemas numéricos, a partir de conceitos mais básicos (no caso, pares
ordenados e relações de equivalência) - é o programa do Hilbert (ou de
Russell e Whitehead), de reduzir toda a matemática à teoria dos conjuntos.
Mas, no fundo, esta é uma construção artificial, ex post.  Pois matemáticos
já usavam todos os números muito antes dessa formalização ser inventada.
E não acho que ela renda muitos frutos, nem pedagógicos (a menos que seu
objetivo seja "entender sem compreender") e nem pra ampliação da fronteira
do conhecimento, exceto colocar os sistemas numéricos numa base axiomática
sólida.
Em particular, no que diz respeito aos números reais, a única coisa que
interessa é que eles são um corpo ordenado completo. Tanto é que vários
livros de análise partem deste axioma e não se preocupam em construir os
reais a partir dos naturais.

[]s,
Claudio.



On Tue, Nov 15, 2022 at 5:07 PM Pedro José  wrote:

> Obrigado a você e ao Cláudio. Mas não sou criativo para inventar. Mas já
> vi que terei que fazer uma homotetia, para as classes de equivalência para
> representar só como um número e não como um par, creio eu.
>
> Cordialmente,
> PJMS
>
> Em ter., 15 de nov. de 2022 às 16:00, Anderson Torres <
> torres.anderson...@gmail.com> escreveu:
>
>>
>>
>> Em ter, 15 de nov de 2022 14:33, Pedro José 
>> escreveu:
>>
>>> Boa tarde!
>>> Para os |Naturais, temos os postulados de Peano.
>>>
>>> Para os Inteiros há alguma formalização?
>>>
>>
>> invente uma!
>>
>> Pode ser por exemplo o conjunto de pares (p,q) tais que p-q é constante.
>>
>> ou melhor (p1,q1)=(p2,q2) se e só se p1+q2=p2+q1.
>>
>>
>>> Acho pobre dizer que é necessário ter outros números devido ao problema
>>> de fechamento nos naturais para a subtração que é fato e daí introduzir os
>>> simétricos que são inteiros e ainda não foram caracterizados.
>>>
>>> No meu antigo ginásio aprendi que os Reais era a união dos conjuntos
>>> disjuntos irracionais e racionais. Os racionais haviam sido bem definidos.
>>> Aí questionei e o que são irracionais? resposta: são os Reais que não são
>>> racionais, os que não podem ser escritos na forma p/q p e q inteiros e
>>> q<>0. Mas me deram um tombo. Definiram os |Reais com base nos irracionais e
>>> os irracionais com base nos |Reais. 3 +2i também não pode ser inscrito na
>>> forma p/q. Só mais tarde no científico, é que meu professor definiu
>>> irracional como um número que não podia ser escrito na forma p/q e cuja
>>> representação decimal tinha uma infinidade de algarismos, sem haver uma
>>> periodicidade.
>>> Na época foi o maior nó que tive com a matemática. O mestre demonstrou
>>> que os racionais eram densos, mas entre eles ainda cabiam os irracionais.
>>> Não satisfeito mostrou que os racionais eram enumeráveis e por absurdo
>>> mostrou que os |Reais não. Não satisfeito mostrou que a cardinalidade do
>>> intervalo [0,1] era maior que a dos |Naturais. Não conseguia conceber que
>>> havia um infinito maior que outro. Outra coisa que demorei a aceitar,mesmo
>>> vendo a bijeção, era que os inteiros e naturais tinham a mesma
>>> cardinalidade. Na minha cabeça, os inteiros têm todos os naturais ainda
>>> sobram os negativos, como é igual?
>>> Hoje, depois de velho, arrumei uma enteada, que muito me pergunta e
>>> estou enrolado. Para dar um ar de superioridade, questionei se conhecia os
>>> inteiros de Gaus, que 5 não era primo nos inteiros de Gaus. Estrepei-me, a
>>> danada foi pesquisar e me questiona sobre o que não tenho um domínio pleno.
>>> Em suma, como apresentei a ela os postulados de Peano para a
>>> caracterização dos Naturais, ela me cobra por algo semelhante para os
>>> Inteiros, e não sei responder.
>>> HELP! SOCORRO! AU SECOURS! AYUDA! AIUTO! HILFE!
>>> Cordialmente,
>>> PJMS
>>>
>>> --
>>> Esta mensagem foi verificada pelo sistema de antivírus e
>>> acredita-se estar livre de perigo.
>>
>>
>> --
>> Esta mensagem foi verificada pelo sistema de antivírus e
>> acredita-se estar livre de perigo.
>
>
> --
> Esta mensagem foi verificada pelo sistema de antivírus e
> acredita-se estar livre de perigo.

-- 
Esta mensagem foi verificada pelo sistema de 

[obm-l] Re: [obm-l] Re: [obm-l] Re: [obm-l] Caracterização de Inteiros

2022-11-15 Por tôpico Anderson Torres
Em ter, 15 de nov de 2022 17:07, Pedro José  escreveu:

> Obrigado a você e ao Cláudio. Mas não sou criativo para inventar. Mas já
> vi que terei que fazer uma homotetia, para as classes de equivalência para
> representar só como um número e não como um par, creio eu.
>

Eu lembro de quando li o Guidorizzi formalizando os reais. Até hoje sinto
que entendo sem compreender, haha!

Por outro lado, números reais (irracionais, no caso) são bem menos
palpáveis que os outros. Dívidas e frações são fáceis de entender, afinal.


> Cordialmente,
> PJMS
>
> Em ter., 15 de nov. de 2022 às 16:00, Anderson Torres <
> torres.anderson...@gmail.com> escreveu:
>
>>
>>
>> Em ter, 15 de nov de 2022 14:33, Pedro José 
>> escreveu:
>>
>>> Boa tarde!
>>> Para os |Naturais, temos os postulados de Peano.
>>>
>>> Para os Inteiros há alguma formalização?
>>>
>>
>> invente uma!
>>
>> Pode ser por exemplo o conjunto de pares (p,q) tais que p-q é constante.
>>
>> ou melhor (p1,q1)=(p2,q2) se e só se p1+q2=p2+q1.
>>
>>
>>> Acho pobre dizer que é necessário ter outros números devido ao problema
>>> de fechamento nos naturais para a subtração que é fato e daí introduzir os
>>> simétricos que são inteiros e ainda não foram caracterizados.
>>>
>>> No meu antigo ginásio aprendi que os Reais era a união dos conjuntos
>>> disjuntos irracionais e racionais. Os racionais haviam sido bem definidos.
>>> Aí questionei e o que são irracionais? resposta: são os Reais que não são
>>> racionais, os que não podem ser escritos na forma p/q p e q inteiros e
>>> q<>0. Mas me deram um tombo. Definiram os |Reais com base nos irracionais e
>>> os irracionais com base nos |Reais. 3 +2i também não pode ser inscrito na
>>> forma p/q. Só mais tarde no científico, é que meu professor definiu
>>> irracional como um número que não podia ser escrito na forma p/q e cuja
>>> representação decimal tinha uma infinidade de algarismos, sem haver uma
>>> periodicidade.
>>> Na época foi o maior nó que tive com a matemática. O mestre demonstrou
>>> que os racionais eram densos, mas entre eles ainda cabiam os irracionais.
>>> Não satisfeito mostrou que os racionais eram enumeráveis e por absurdo
>>> mostrou que os |Reais não. Não satisfeito mostrou que a cardinalidade do
>>> intervalo [0,1] era maior que a dos |Naturais. Não conseguia conceber que
>>> havia um infinito maior que outro. Outra coisa que demorei a aceitar,mesmo
>>> vendo a bijeção, era que os inteiros e naturais tinham a mesma
>>> cardinalidade. Na minha cabeça, os inteiros têm todos os naturais ainda
>>> sobram os negativos, como é igual?
>>> Hoje, depois de velho, arrumei uma enteada, que muito me pergunta e
>>> estou enrolado. Para dar um ar de superioridade, questionei se conhecia os
>>> inteiros de Gaus, que 5 não era primo nos inteiros de Gaus. Estrepei-me, a
>>> danada foi pesquisar e me questiona sobre o que não tenho um domínio pleno.
>>> Em suma, como apresentei a ela os postulados de Peano para a
>>> caracterização dos Naturais, ela me cobra por algo semelhante para os
>>> Inteiros, e não sei responder.
>>> HELP! SOCORRO! AU SECOURS! AYUDA! AIUTO! HILFE!
>>> Cordialmente,
>>> PJMS
>>>
>>> --
>>> Esta mensagem foi verificada pelo sistema de antivírus e
>>> acredita-se estar livre de perigo.
>>
>>
>> --
>> Esta mensagem foi verificada pelo sistema de antivírus e
>> acredita-se estar livre de perigo.
>
>
> --
> Esta mensagem foi verificada pelo sistema de antivírus e
> acredita-se estar livre de perigo.

-- 
Esta mensagem foi verificada pelo sistema de antiv�rus e
 acredita-se estar livre de perigo.



[obm-l] Re: [obm-l] Re: [obm-l] Caracterização de Inteiros

2022-11-15 Por tôpico Pedro José
Obrigado a você e ao Cláudio. Mas não sou criativo para inventar. Mas já vi
que terei que fazer uma homotetia, para as classes de equivalência para
representar só como um número e não como um par, creio eu.

Cordialmente,
PJMS

Em ter., 15 de nov. de 2022 às 16:00, Anderson Torres <
torres.anderson...@gmail.com> escreveu:

>
>
> Em ter, 15 de nov de 2022 14:33, Pedro José 
> escreveu:
>
>> Boa tarde!
>> Para os |Naturais, temos os postulados de Peano.
>>
>> Para os Inteiros há alguma formalização?
>>
>
> invente uma!
>
> Pode ser por exemplo o conjunto de pares (p,q) tais que p-q é constante.
>
> ou melhor (p1,q1)=(p2,q2) se e só se p1+q2=p2+q1.
>
>
>> Acho pobre dizer que é necessário ter outros números devido ao problema
>> de fechamento nos naturais para a subtração que é fato e daí introduzir os
>> simétricos que são inteiros e ainda não foram caracterizados.
>>
>> No meu antigo ginásio aprendi que os Reais era a união dos conjuntos
>> disjuntos irracionais e racionais. Os racionais haviam sido bem definidos.
>> Aí questionei e o que são irracionais? resposta: são os Reais que não são
>> racionais, os que não podem ser escritos na forma p/q p e q inteiros e
>> q<>0. Mas me deram um tombo. Definiram os |Reais com base nos irracionais e
>> os irracionais com base nos |Reais. 3 +2i também não pode ser inscrito na
>> forma p/q. Só mais tarde no científico, é que meu professor definiu
>> irracional como um número que não podia ser escrito na forma p/q e cuja
>> representação decimal tinha uma infinidade de algarismos, sem haver uma
>> periodicidade.
>> Na época foi o maior nó que tive com a matemática. O mestre demonstrou
>> que os racionais eram densos, mas entre eles ainda cabiam os irracionais.
>> Não satisfeito mostrou que os racionais eram enumeráveis e por absurdo
>> mostrou que os |Reais não. Não satisfeito mostrou que a cardinalidade do
>> intervalo [0,1] era maior que a dos |Naturais. Não conseguia conceber que
>> havia um infinito maior que outro. Outra coisa que demorei a aceitar,mesmo
>> vendo a bijeção, era que os inteiros e naturais tinham a mesma
>> cardinalidade. Na minha cabeça, os inteiros têm todos os naturais ainda
>> sobram os negativos, como é igual?
>> Hoje, depois de velho, arrumei uma enteada, que muito me pergunta e estou
>> enrolado. Para dar um ar de superioridade, questionei se conhecia os
>> inteiros de Gaus, que 5 não era primo nos inteiros de Gaus. Estrepei-me, a
>> danada foi pesquisar e me questiona sobre o que não tenho um domínio pleno.
>> Em suma, como apresentei a ela os postulados de Peano para a
>> caracterização dos Naturais, ela me cobra por algo semelhante para os
>> Inteiros, e não sei responder.
>> HELP! SOCORRO! AU SECOURS! AYUDA! AIUTO! HILFE!
>> Cordialmente,
>> PJMS
>>
>> --
>> Esta mensagem foi verificada pelo sistema de antivírus e
>> acredita-se estar livre de perigo.
>
>
> --
> Esta mensagem foi verificada pelo sistema de antivírus e
> acredita-se estar livre de perigo.

-- 
Esta mensagem foi verificada pelo sistema de antiv�rus e
 acredita-se estar livre de perigo.



[obm-l] Re: [obm-l] Caracterização de Inteiros

2022-11-15 Por tôpico Anderson Torres
Em ter, 15 de nov de 2022 14:33, Pedro José  escreveu:

> Boa tarde!
> Para os |Naturais, temos os postulados de Peano.
>
> Para os Inteiros há alguma formalização?
>

invente uma!

Pode ser por exemplo o conjunto de pares (p,q) tais que p-q é constante.

ou melhor (p1,q1)=(p2,q2) se e só se p1+q2=p2+q1.


> Acho pobre dizer que é necessário ter outros números devido ao problema de
> fechamento nos naturais para a subtração que é fato e daí introduzir os
> simétricos que são inteiros e ainda não foram caracterizados.
>
> No meu antigo ginásio aprendi que os Reais era a união dos conjuntos
> disjuntos irracionais e racionais. Os racionais haviam sido bem definidos.
> Aí questionei e o que são irracionais? resposta: são os Reais que não são
> racionais, os que não podem ser escritos na forma p/q p e q inteiros e
> q<>0. Mas me deram um tombo. Definiram os |Reais com base nos irracionais e
> os irracionais com base nos |Reais. 3 +2i também não pode ser inscrito na
> forma p/q. Só mais tarde no científico, é que meu professor definiu
> irracional como um número que não podia ser escrito na forma p/q e cuja
> representação decimal tinha uma infinidade de algarismos, sem haver uma
> periodicidade.
> Na época foi o maior nó que tive com a matemática. O mestre demonstrou que
> os racionais eram densos, mas entre eles ainda cabiam os irracionais. Não
> satisfeito mostrou que os racionais eram enumeráveis e por absurdo mostrou
> que os |Reais não. Não satisfeito mostrou que a cardinalidade do intervalo
> [0,1] era maior que a dos |Naturais. Não conseguia conceber que havia um
> infinito maior que outro. Outra coisa que demorei a aceitar,mesmo vendo a
> bijeção, era que os inteiros e naturais tinham a mesma cardinalidade. Na
> minha cabeça, os inteiros têm todos os naturais ainda sobram os negativos,
> como é igual?
> Hoje, depois de velho, arrumei uma enteada, que muito me pergunta e estou
> enrolado. Para dar um ar de superioridade, questionei se conhecia os
> inteiros de Gaus, que 5 não era primo nos inteiros de Gaus. Estrepei-me, a
> danada foi pesquisar e me questiona sobre o que não tenho um domínio pleno.
> Em suma, como apresentei a ela os postulados de Peano para a
> caracterização dos Naturais, ela me cobra por algo semelhante para os
> Inteiros, e não sei responder.
> HELP! SOCORRO! AU SECOURS! AYUDA! AIUTO! HILFE!
> Cordialmente,
> PJMS
>
> --
> Esta mensagem foi verificada pelo sistema de antivírus e
> acredita-se estar livre de perigo.

-- 
Esta mensagem foi verificada pelo sistema de antiv�rus e
 acredita-se estar livre de perigo.



Re: [obm-l] Caracterização de Inteiros

2022-11-15 Por tôpico Claudio Buffara
A única que conheço e’ a que define uma relação de equivalência em pares 
ordenados de naturais (união {0}) dada por (a,b) ~ (c,d)  <==> a+d = b+c. Os 
inteiros são as classes de equivalência desta relação.

Enviado do meu iPhone

> Em 15 de nov. de 2022, à(s) 14:33, Pedro José  escreveu:
> 
> 
> Boa tarde!
> Para os |Naturais, temos os postulados de Peano.
> 
> Para os Inteiros há alguma formalização?
> 
> Acho pobre dizer que é necessário ter outros números devido ao problema de 
> fechamento nos naturais para a subtração que é fato e daí introduzir os 
> simétricos que são inteiros e ainda não foram caracterizados. 
> 
> No meu antigo ginásio aprendi que os Reais era a união dos conjuntos 
> disjuntos irracionais e racionais. Os racionais haviam sido bem definidos. 
> Aí questionei e o que são irracionais? resposta: são os Reais que não 
> são racionais, os que não podem ser escritos na forma p/q p e q inteiros e 
> q<>0. Mas me deram um tombo. Definiram os |Reais com base nos irracionais e 
> os irracionais com base nos |Reais. 3 +2i também não pode ser inscrito na 
> forma p/q. Só mais tarde no científico, é que meu professor definiu 
> irracional como um número que não podia ser escrito na forma p/q e cuja 
> representação decimal tinha uma infinidade de algarismos, sem haver uma 
> periodicidade.
> Na época foi o maior nó que tive com a matemática. O mestre demonstrou que 
> os racionais eram densos, mas entre eles ainda cabiam os irracionais. Não 
> satisfeito mostrou que os racionais eram enumeráveis e por absurdo mostrou 
> que os |Reais não. Não satisfeito mostrou que a cardinalidade do intervalo 
> [0,1] era maior que a dos |Naturais. Não conseguia conceber que havia um 
> infinito maior que outro. Outra coisa que demorei a aceitar,mesmo vendo a 
> bijeção, era que os inteiros e naturais tinham a mesma cardinalidade. Na 
> minha cabeça, os inteiros têm todos os naturais ainda sobram os negativos, 
> como é igual?
> Hoje, depois de velho, arrumei uma enteada, que muito me pergunta e estou 
> enrolado. Para dar um ar de superioridade, questionei se conhecia os inteiros 
> de Gaus, que 5 não era primo nos inteiros de Gaus. Estrepei-me, a danada foi 
> pesquisar e me questiona sobre o que não tenho um domínio pleno.
> Em suma, como apresentei a ela os postulados de Peano para a caracterização 
> dos Naturais, ela me cobra por algo semelhante para os Inteiros, e não sei 
> responder.
> HELP! SOCORRO! AU SECOURS! AYUDA! AIUTO! HILFE!
> Cordialmente,
> PJMS
> 
> -- 
> Esta mensagem foi verificada pelo sistema de antivírus e 
> acredita-se estar livre de perigo.

-- 
Esta mensagem foi verificada pelo sistema de antiv�rus e
 acredita-se estar livre de perigo.


=
Instru��es para entrar na lista, sair da lista e usar a lista em
http://www.mat.puc-rio.br/~obmlistas/obm-l.html
=


[obm-l] Caracterização de Inteiros

2022-11-15 Por tôpico Pedro José
Boa tarde!
Para os |Naturais, temos os postulados de Peano.

Para os Inteiros há alguma formalização?

Acho pobre dizer que é necessário ter outros números devido ao problema de
fechamento nos naturais para a subtração que é fato e daí introduzir os
simétricos que são inteiros e ainda não foram caracterizados.

No meu antigo ginásio aprendi que os Reais era a união dos conjuntos
disjuntos irracionais e racionais. Os racionais haviam sido bem definidos.
Aí questionei e o que são irracionais? resposta: são os Reais que não são
racionais, os que não podem ser escritos na forma p/q p e q inteiros e
q<>0. Mas me deram um tombo. Definiram os |Reais com base nos irracionais e
os irracionais com base nos |Reais. 3 +2i também não pode ser inscrito na
forma p/q. Só mais tarde no científico, é que meu professor definiu
irracional como um número que não podia ser escrito na forma p/q e cuja
representação decimal tinha uma infinidade de algarismos, sem haver uma
periodicidade.
Na época foi o maior nó que tive com a matemática. O mestre demonstrou que
os racionais eram densos, mas entre eles ainda cabiam os irracionais. Não
satisfeito mostrou que os racionais eram enumeráveis e por absurdo mostrou
que os |Reais não. Não satisfeito mostrou que a cardinalidade do intervalo
[0,1] era maior que a dos |Naturais. Não conseguia conceber que havia um
infinito maior que outro. Outra coisa que demorei a aceitar,mesmo vendo a
bijeção, era que os inteiros e naturais tinham a mesma cardinalidade. Na
minha cabeça, os inteiros têm todos os naturais ainda sobram os negativos,
como é igual?
Hoje, depois de velho, arrumei uma enteada, que muito me pergunta e estou
enrolado. Para dar um ar de superioridade, questionei se conhecia os
inteiros de Gaus, que 5 não era primo nos inteiros de Gaus. Estrepei-me, a
danada foi pesquisar e me questiona sobre o que não tenho um domínio pleno.
Em suma, como apresentei a ela os postulados de Peano para a caracterização
dos Naturais, ela me cobra por algo semelhante para os Inteiros, e não sei
responder.
HELP! SOCORRO! AU SECOURS! AYUDA! AIUTO! HILFE!
Cordialmente,
PJMS

-- 
Esta mensagem foi verificada pelo sistema de antiv�rus e
 acredita-se estar livre de perigo.



[obm-l] matemática recreativa

2022-11-14 Por tôpico Daniel Jelin
amigos, além do prof. melo e souza, o grande malba tahan, sabem dizer que
outros nomes contribuíram para a matemática recreativa no brasil?

-- 
Esta mensagem foi verificada pelo sistema de antiv�rus e
 acredita-se estar livre de perigo.



Re: [obm-l] problema de probabilidade

2022-11-09 Por tôpico Claudio Buffara
Essa também:
https://thedailyviz.com/2016/09/17/how-common-is-your-birthday-dailyviz/


On Wed, Nov 9, 2022 at 12:04 PM Claudio Buffara 
wrote:

> Achei isso aqui interessante: https://www.panix.com/~murphy/bday.html
>
> []s,
> Claudio.
>
> On Tue, Nov 8, 2022 at 9:56 PM Ralph Costa Teixeira 
> wrote:
>
>> Mis ou menos... O que faltou foi a hipótese exata da distribuição de
>> probabilidade dos aniversários.
>>
>> Se a gente supõe que cada mês tem os mesmos 1/12 de chance para cada
>> aluno, e que os meses são independentes entre si, sim,
>> p=12/12^2=1/12~8.3%.
>>
>> Agora, talvez um modelo um pouco mais preciso seria supor que cada DIA do
>> ano tem a mesma probabilidade (e que são independentes entre si). Isto
>> afeta um tiquinho a resposta, porque cada mes têm um número ligeiramente
>> diferente de dias! Ignorando anos bissextos (huh!?!), temos:
>> -- 7 meses com 31 dias;
>> -- 4 meses com 30 dias;
>> -- 1 mes com 28 dias;
>> Portanto, seria um pouco mais "realista" usar:
>> p=(7*31^2+4*30^2+28^2)/(365^2) ~ 8.34003%
>>
>> Eu ponho esse "realista" bem entre aspas; primeiro, porque eu ignorei
>> anos bissextos (fique à vontade para inclui-los e refazer a conta :D :D
>> :D); mas a hipótese de que todos os dias do ano tem a mesma probabilidade
>> não é tão realista quanto parece! Existe uma certa "concentração" de
>> aniversários em determinadas épocas do ano... mas, sem dados exatos sobre
>> como seja a tal concentração, o melhor que podemos fazer seria uma das
>> estimativas acima.
>>
>> Ainda tem um segundo problema sutil: *mesmo que todos os dias tivessem a
>> mesma probabilidade, talvez n*ã*o seja 100% correto supor que os
>> aniversários dos alunos da mesma turma do CMBel sejam independentes*!
>> Por exemplo, existe uma probabilidade maior que zero de ter gêmeos numa
>> mesma turma (comum uma família com gêmeos colocá-los na mesma escola), o
>> que afeta a independência dos dados, e muda um pouquinho aqueles 8.3% (para
>> cima)... sem uma estimativa desta probabilidade de ter gêmeos na mesma
>> turma, não conseguimos calcular a resposta "exata".
>>
>> Isto tudo dito... em quase qualquer problema de probabilidade a gente vai
>> ter que fazer ALGUMA hipótese simplificadora para poder sair do lugar.
>> Assim, eu diria que o problema não está 100% bem posto, mas não acho
>> ridículo fazer uma das hipóteses simplificadoras acima que levam a 8.3%
>> ou 8.34003% (e a diferença me parece tão pequena que eu aceitaria ambas as
>> respostas como corretas, desde que as hipóteses utilizadas em cada caso
>> fossem citadas).
>>
>> Abraço, Ralph.
>>
>> On Tue, Nov 8, 2022 at 3:07 PM Luis Paulo  wrote:
>>
>>> Prezados, o problema abaixo está bem posto?
>>>
>>> Uma turma do CMBel tem 25 alunos. Escolhendo-se aleatoriamente dois
>>> estudantes dessa turma, qual a probabilidade de eles façam aniversário no
>>> mesmo mês?
>>>
>>> A resposta da banca: 1/12.
>>>
>>>
>>>
>>> --
>>> Esta mensagem foi verificada pelo sistema de antivírus e
>>> acredita-se estar livre de perigo.
>>
>>
>> --
>> Esta mensagem foi verificada pelo sistema de antivírus e
>> acredita-se estar livre de perigo.
>
>

-- 
Esta mensagem foi verificada pelo sistema de antiv�rus e
 acredita-se estar livre de perigo.



Re: [obm-l] problema de probabilidade

2022-11-09 Por tôpico Claudio Buffara
Achei isso aqui interessante: https://www.panix.com/~murphy/bday.html

[]s,
Claudio.

On Tue, Nov 8, 2022 at 9:56 PM Ralph Costa Teixeira 
wrote:

> Mis ou menos... O que faltou foi a hipótese exata da distribuição de
> probabilidade dos aniversários.
>
> Se a gente supõe que cada mês tem os mesmos 1/12 de chance para cada
> aluno, e que os meses são independentes entre si, sim,
> p=12/12^2=1/12~8.3%.
>
> Agora, talvez um modelo um pouco mais preciso seria supor que cada DIA do
> ano tem a mesma probabilidade (e que são independentes entre si). Isto
> afeta um tiquinho a resposta, porque cada mes têm um número ligeiramente
> diferente de dias! Ignorando anos bissextos (huh!?!), temos:
> -- 7 meses com 31 dias;
> -- 4 meses com 30 dias;
> -- 1 mes com 28 dias;
> Portanto, seria um pouco mais "realista" usar:
> p=(7*31^2+4*30^2+28^2)/(365^2) ~ 8.34003%
>
> Eu ponho esse "realista" bem entre aspas; primeiro, porque eu ignorei
> anos bissextos (fique à vontade para inclui-los e refazer a conta :D :D
> :D); mas a hipótese de que todos os dias do ano tem a mesma probabilidade
> não é tão realista quanto parece! Existe uma certa "concentração" de
> aniversários em determinadas épocas do ano... mas, sem dados exatos sobre
> como seja a tal concentração, o melhor que podemos fazer seria uma das
> estimativas acima.
>
> Ainda tem um segundo problema sutil: *mesmo que todos os dias tivessem a
> mesma probabilidade, talvez n*ã*o seja 100% correto supor que os
> aniversários dos alunos da mesma turma do CMBel sejam independentes*! Por
> exemplo, existe uma probabilidade maior que zero de ter gêmeos numa mesma
> turma (comum uma família com gêmeos colocá-los na mesma escola), o que
> afeta a independência dos dados, e muda um pouquinho aqueles 8.3% (para
> cima)... sem uma estimativa desta probabilidade de ter gêmeos na mesma
> turma, não conseguimos calcular a resposta "exata".
>
> Isto tudo dito... em quase qualquer problema de probabilidade a gente vai
> ter que fazer ALGUMA hipótese simplificadora para poder sair do lugar.
> Assim, eu diria que o problema não está 100% bem posto, mas não acho
> ridículo fazer uma das hipóteses simplificadoras acima que levam a 8.3%
> ou 8.34003% (e a diferença me parece tão pequena que eu aceitaria ambas as
> respostas como corretas, desde que as hipóteses utilizadas em cada caso
> fossem citadas).
>
> Abraço, Ralph.
>
> On Tue, Nov 8, 2022 at 3:07 PM Luis Paulo  wrote:
>
>> Prezados, o problema abaixo está bem posto?
>>
>> Uma turma do CMBel tem 25 alunos. Escolhendo-se aleatoriamente dois
>> estudantes dessa turma, qual a probabilidade de eles façam aniversário no
>> mesmo mês?
>>
>> A resposta da banca: 1/12.
>>
>>
>>
>> --
>> Esta mensagem foi verificada pelo sistema de antivírus e
>> acredita-se estar livre de perigo.
>
>
> --
> Esta mensagem foi verificada pelo sistema de antivírus e
> acredita-se estar livre de perigo.

-- 
Esta mensagem foi verificada pelo sistema de antiv�rus e
 acredita-se estar livre de perigo.



Re: [obm-l] problema de probabilidade

2022-11-09 Por tôpico Anderson Torres
Em ter, 8 de nov de 2022 21:55, Ralph Costa Teixeira 
escreveu:

> Mis ou menos... O que faltou foi a hipótese exata da distribuição de
> probabilidade dos aniversários.
>
> Se a gente supõe que cada mês tem os mesmos 1/12 de chance para cada
> aluno, e que os meses são independentes entre si, sim,
> p=12/12^2=1/12~8.3%.
>
> Agora, talvez um modelo um pouco mais preciso seria supor que cada DIA do
> ano tem a mesma probabilidade (e que são independentes entre si). Isto
> afeta um tiquinho a resposta, porque cada mes têm um número ligeiramente
> diferente de dias! Ignorando anos bissextos (huh!?!), temos:
> -- 7 meses com 31 dias;
> -- 4 meses com 30 dias;
> -- 1 mes com 28 dias;
> Portanto, seria um pouco mais "realista" usar:
> p=(7*31^2+4*30^2+28^2)/(365^2) ~ 8.34003%
>
> Eu ponho esse "realista" bem entre aspas; primeiro, porque eu ignorei
> anos bissextos (fique à vontade para inclui-los e refazer a conta :D :D
> :D); mas a hipótese de que todos os dias do ano tem a mesma probabilidade
> não é tão realista quanto parece! Existe uma certa "concentração" de
> aniversários em determinadas épocas do ano... mas, sem dados exatos sobre
> como seja a tal concentração, o melhor que podemos fazer seria uma das
> estimativas acima.
>

Em uma turma com tão pouca gente, eu acho que considerações como "a
concentração de pessoas concebidas no Carnaval" podem ser ignoradas para um
problema tão simples. E, pelo que se nota, a conta mais limpa dá uma
diferença minúscula, 0,01%. Desconheço aplicação tão precisa na prática.


> Ainda tem um segundo problema sutil: *mesmo que todos os dias tivessem a
> mesma probabilidade, talvez n*ã*o seja 100% correto supor que os
> aniversários dos alunos da mesma turma do CMBel sejam independentes*! Por
> exemplo, existe uma probabilidade maior que zero de ter gêmeos numa mesma
> turma (comum uma família com gêmeos colocá-los na mesma escola), o que
> afeta a independência dos dados, e muda um pouquinho aqueles 8.3% (para
> cima)... sem uma estimativa desta probabilidade de ter gêmeos na mesma
> turma, não conseguimos calcular a resposta "exata".
>
> Isto tudo dito... em quase qualquer problema de probabilidade a gente vai
> ter que fazer ALGUMA hipótese simplificadora para poder sair do lugar.
> Assim, eu diria que o problema não está 100% bem posto, mas não acho
> ridículo fazer uma das hipóteses simplificadoras acima que levam a 8.3%
> ou 8.34003% (e a diferença me parece tão pequena que eu aceitaria ambas as
> respostas como corretas, desde que as hipóteses utilizadas em cada caso
> fossem citadas).
>
> Abraço, Ralph.
>
> On Tue, Nov 8, 2022 at 3:07 PM Luis Paulo  wrote:
>
>> Prezados, o problema abaixo está bem posto?
>>
>> Uma turma do CMBel tem 25 alunos. Escolhendo-se aleatoriamente dois
>> estudantes dessa turma, qual a probabilidade de eles façam aniversário no
>> mesmo mês?
>>
>> A resposta da banca: 1/12.
>>
>>
>>
>> --
>> Esta mensagem foi verificada pelo sistema de antivírus e
>> acredita-se estar livre de perigo.
>
>
> --
> Esta mensagem foi verificada pelo sistema de antivírus e
> acredita-se estar livre de perigo.

-- 
Esta mensagem foi verificada pelo sistema de antiv�rus e
 acredita-se estar livre de perigo.



Re: [obm-l] problema de probabilidade

2022-11-08 Por tôpico Ralph Costa Teixeira
Mis ou menos... O que faltou foi a hipótese exata da distribuição de
probabilidade dos aniversários.

Se a gente supõe que cada mês tem os mesmos 1/12 de chance para cada aluno,
e que os meses são independentes entre si, sim, p=12/12^2=1/12~8.3%.

Agora, talvez um modelo um pouco mais preciso seria supor que cada DIA do
ano tem a mesma probabilidade (e que são independentes entre si). Isto
afeta um tiquinho a resposta, porque cada mes têm um número ligeiramente
diferente de dias! Ignorando anos bissextos (huh!?!), temos:
-- 7 meses com 31 dias;
-- 4 meses com 30 dias;
-- 1 mes com 28 dias;
Portanto, seria um pouco mais "realista" usar:
p=(7*31^2+4*30^2+28^2)/(365^2) ~ 8.34003%

Eu ponho esse "realista" bem entre aspas; primeiro, porque eu ignorei
anos bissextos (fique à vontade para inclui-los e refazer a conta :D :D
:D); mas a hipótese de que todos os dias do ano tem a mesma probabilidade
não é tão realista quanto parece! Existe uma certa "concentração" de
aniversários em determinadas épocas do ano... mas, sem dados exatos sobre
como seja a tal concentração, o melhor que podemos fazer seria uma das
estimativas acima.

Ainda tem um segundo problema sutil: *mesmo que todos os dias tivessem a
mesma probabilidade, talvez n*ã*o seja 100% correto supor que os
aniversários dos alunos da mesma turma do CMBel sejam independentes*! Por
exemplo, existe uma probabilidade maior que zero de ter gêmeos numa mesma
turma (comum uma família com gêmeos colocá-los na mesma escola), o que
afeta a independência dos dados, e muda um pouquinho aqueles 8.3% (para
cima)... sem uma estimativa desta probabilidade de ter gêmeos na mesma
turma, não conseguimos calcular a resposta "exata".

Isto tudo dito... em quase qualquer problema de probabilidade a gente vai
ter que fazer ALGUMA hipótese simplificadora para poder sair do lugar.
Assim, eu diria que o problema não está 100% bem posto, mas não acho
ridículo fazer uma das hipóteses simplificadoras acima que levam a 8.3%
ou 8.34003% (e a diferença me parece tão pequena que eu aceitaria ambas as
respostas como corretas, desde que as hipóteses utilizadas em cada caso
fossem citadas).

Abraço, Ralph.

On Tue, Nov 8, 2022 at 3:07 PM Luis Paulo  wrote:

> Prezados, o problema abaixo está bem posto?
>
> Uma turma do CMBel tem 25 alunos. Escolhendo-se aleatoriamente dois
> estudantes dessa turma, qual a probabilidade de eles façam aniversário no
> mesmo mês?
>
> A resposta da banca: 1/12.
>
>
>
> --
> Esta mensagem foi verificada pelo sistema de antivírus e
> acredita-se estar livre de perigo.

-- 
Esta mensagem foi verificada pelo sistema de antiv�rus e
 acredita-se estar livre de perigo.



[obm-l] problema de probabilidade

2022-11-08 Por tôpico Luis Paulo
Prezados, o problema abaixo está bem posto?Uma turma do CMBel tem 25 alunos. Escolhendo-se aleatoriamente dois estudantes dessa turma, qual a probabilidade de eles façam aniversário no mesmo mês?A resposta da banca: 1/12.--
Esta mensagem foi verificada pelo sistema de antivírus e 
 acredita-se estar livre de perigo.




[obm-l] construir triângulos dados

2022-10-09 Por tôpico Luís Lopes
Sauda,c~oes,

O problema terá duas soluções (h_a=h=altura e d_a=d=bissetriz interna, k=d/h) se
0https://www.wolframalpha.com/input?i=0%3CB%3C%3Dpi%2F3%3B+1%3Ck%3C%5Csec%28B%2F2%29%3B+tan%5E2B%3D4%28k%5E2-1%29%2F%28k%5E4-4k%5E2%2B4%29

https://www.wolframalpha.com/input?i=pi%2F3%3CB%3Cpi%2F2%3B+1%3Ck%3C%5Ccsc%28B%29%3B+tan%5E2B%3D4%28k%5E2-1%29%2F%28k%5E4-4k%5E2%2B4%29

Obrigado.
Luís



-- 
Esta mensagem foi verificada pelo sistema de antivírus e
 acredita-se estar livre de perigo.



Re: [obm-l] integrais

2022-10-05 Por tôpico carlos h Souza
Desculpe -me ao colar o texto do bloco de notas, acabei verificando que
havia uma imagem. Vou refazer a pergunta ..
 Obrigado

Em qua, 5 de out de 2022 16:24, Anderson Torres <
torres.anderson...@gmail.com> escreveu:

> É spam??
>
> Em ter, 4 de out de 2022 15:48, carlos h Souza 
> escreveu:
>
>>
> --
> Esta mensagem foi verificada pelo sistema de antivírus e
> acredita-se estar livre de perigo.

-- 
Esta mensagem foi verificada pelo sistema de antiv�rus e
 acredita-se estar livre de perigo.



Re: [obm-l] integrais

2022-10-05 Por tôpico Anderson Torres
É spam??

Em ter, 4 de out de 2022 15:48, carlos h Souza 
escreveu:

>

-- 
Esta mensagem foi verificada pelo sistema de antiv�rus e
 acredita-se estar livre de perigo.



[obm-l] integrais

2022-10-04 Por tôpico carlos h Souza


[obm-l] Fatoração

2022-09-14 Por tôpico Daniel Quevedo
A soma dos valores inteiros de a para os quais (x -10)(x+a) +1 seja
faturável num produto (x+b)(x+c) com b e c inteiros é:

A) 8
B) 10
C) 12
D) 20
E) 24

Resp: D
-- 
Fiscal: Daniel Quevedo

-- 
Esta mensagem foi verificada pelo sistema de antiv�rus e
 acredita-se estar livre de perigo.



[obm-l] Re: [obm-l] Sugestões para Iniciação Criança

2022-09-14 Por tôpico Tiago Machado
Não sei como andam as atualizações, mas talvez a revista Eureka seja um dos
bons lugares para começar
https://www.obm.org.br/revista-eureka/

On Wed, Sep 14, 2022 at 10:49 AM Esaú Gomes  wrote:

> Olá pessoal, tenho um filho de 10 anos (quinto ano) e ele curte
> matemática. Gostaria de iniciá-lo em material referente às olimpíadas para
> ver se ele pega gosto. Alguém indica material para fazer sua iniciação?
>
> Desde já, obrigado.
>
> --
> Esta mensagem foi verificada pelo sistema de antivírus e
> acredita-se estar livre de perigo.

-- 
Esta mensagem foi verificada pelo sistema de antiv�rus e
 acredita-se estar livre de perigo.



[obm-l] Sugestões para Iniciação Criança

2022-09-14 Por tôpico Esaú Gomes
Olá pessoal, tenho um filho de 10 anos (quinto ano) e ele curte matemática.
Gostaria de iniciá-lo em material referente às olimpíadas para ver se ele
pega gosto. Alguém indica material para fazer sua iniciação?

Desde já, obrigado.

-- 
Esta mensagem foi verificada pelo sistema de antiv�rus e
 acredita-se estar livre de perigo.



[obm-l] Re: [obm-l] Perguntas mínimas no Tabuleiro

2022-09-14 Por tôpico Anderson Torres
Em ter, 13 de set de 2022 22:59, Jeferson Almir 
escreveu:

> Os números de 1 a 49 são arbitrariamente dispostos num tabuleiro quadrado
> 7x7 . Podemos escolher qualquer quadrado composto de múltiplas células e
> perguntar quais números estão contidos nele. Ao menos quantas perguntas são
> necessárias para determinarmos a configuração exata dos números?
>
> Alguém tem uma ideia ótima ??
>

tenho não, mas para o caso 3x3 a resposta é no máximo 4. Perguntando quem
são os elementos dos quadradinhos 2x2, é possível obter quem são os 9 caras
usando uniões e intersecções.

com isso já podemos recursivamente cortar o quadrado 7x7 em quadrados 3x3.
Se fosse 9x9, isso daria no máximo 24.

vou pensar mais em casa.


>
> --
> Esta mensagem foi verificada pelo sistema de antivírus e
> acredita-se estar livre de perigo.

-- 
Esta mensagem foi verificada pelo sistema de antiv�rus e
 acredita-se estar livre de perigo.



[obm-l] Perguntas mínimas no Tabuleiro

2022-09-13 Por tôpico Jeferson Almir
Os números de 1 a 49 são arbitrariamente dispostos num tabuleiro quadrado
7x7 . Podemos escolher qualquer quadrado composto de múltiplas células e
perguntar quais números estão contidos nele. Ao menos quantas perguntas são
necessárias para determinarmos a configuração exata dos números?

Alguém tem uma ideia ótima ??

-- 
Esta mensagem foi verificada pelo sistema de antiv�rus e
 acredita-se estar livre de perigo.



[obm-l] Re: [obm-l] Séries

2022-08-25 Por tôpico Esdras Muniz
Me manda.

Em qui, 25 de ago de 2022 17:36, Israel Meireles Chrisostomo <
israelmchrisost...@gmail.com> escreveu:

> Olá pessoal, recentemente eu tive umas ideias sobre séries envolvendo o
> número e (napier), o seno e o cosseno.Alguém por favor poderia me
> corrigir?São ideias originais e séries infinitas nunca antes pensadas.
> Alguém por favor me ajuda a corrigir.Ver se estou viajandoMeu desejo é
> que vcs digam que esteja certo, sejam pacientes por favor.Quem se dispor,
> por favor, chama inbox
>
> -
> Somente a Deus Glória.
> Israel Meireles Chrisostomo
>
> --
> Esta mensagem foi verificada pelo sistema de antivírus e
> acredita-se estar livre de perigo.

-- 
Esta mensagem foi verificada pelo sistema de antiv�rus e
 acredita-se estar livre de perigo.



[obm-l] Séries

2022-08-25 Por tôpico Israel Meireles Chrisostomo
Olá pessoal, recentemente eu tive umas ideias sobre séries envolvendo o
número e (napier), o seno e o cosseno.Alguém por favor poderia me
corrigir?São ideias originais e séries infinitas nunca antes pensadas.
Alguém por favor me ajuda a corrigir.Ver se estou viajandoMeu desejo é
que vcs digam que esteja certo, sejam pacientes por favor.Quem se dispor,
por favor, chama inbox

-
Somente a Deus Glória.
Israel Meireles Chrisostomo

-- 
Esta mensagem foi verificada pelo sistema de antiv�rus e
 acredita-se estar livre de perigo.



[obm-l] Re: [obm-l] Re: [obm-l] Fwd: Módulo

2022-08-14 Por tôpico Rubens Vilhena Fonseca
Uma coisa que você deve definir é a paridade de n. Vamos reescrever em
linguagem de congruências :
2^n==1 (mod 3). Sabendo que 2== -1 (mod 3), então  (-1)^n == 1 (mod 3). O
que só será verdade se n for par.
Então, para n = 2k, temos 4^k = 3x +1.  Por experimentação, você pode
concluir alguns pares (k, x) de solução, (0, 0);
(1, 1); (2, 5); (3, 21)...Então, seu trabalho é mostrar que o par (k,
(4^k-1)/3 ) é uma solução.

Em qui., 11 de ago. de 2022 às 17:38, Anderson Torres <
torres.anderson...@gmail.com> escreveu:

>
>
> Em qui, 11 de ago de 2022 16:12, Esaú Gomes 
> escreveu:
>
>> Alguém poderia me falar o que estudar mais especificamente na questão
>> abaixo?
>>
>> Para quais valores naturais  de *n* e *x*, existe solução
>> 2^n = 3x + 1.
>>
>
>   Provas antigas.
>
> Esses problemas são resolvidos geralmente apelando para fatos padrão de
> congruências, em especial potenciação, ordem etc.
>
> E, no geral, a melhor maneira de entender e aplicar estes fatos é mediante
> treino, treino e mais treino.
>
> --
>> Esta mensagem foi verificada pelo sistema de antivírus e
>> acredita-se estar livre de perigo.
>
>
> --
> Esta mensagem foi verificada pelo sistema de antivírus e
> acredita-se estar livre de perigo.

-- 
Esta mensagem foi verificada pelo sistema de antiv�rus e
 acredita-se estar livre de perigo.



[obm-l] Re: [obm-l] Fwd: Módulo

2022-08-11 Por tôpico Anderson Torres
Em qui, 11 de ago de 2022 16:12, Esaú Gomes  escreveu:

> Alguém poderia me falar o que estudar mais especificamente na questão
> abaixo?
>
> Para quais valores naturais  de *n* e *x*, existe solução
> 2^n = 3x + 1.
>

  Provas antigas.

Esses problemas são resolvidos geralmente apelando para fatos padrão de
congruências, em especial potenciação, ordem etc.

E, no geral, a melhor maneira de entender e aplicar estes fatos é mediante
treino, treino e mais treino.

-- 
> Esta mensagem foi verificada pelo sistema de antivírus e
> acredita-se estar livre de perigo.

-- 
Esta mensagem foi verificada pelo sistema de antiv�rus e
 acredita-se estar livre de perigo.



[obm-l] Re: [obm-l] Fwd: Módulo

2022-08-11 Por tôpico Carlos Gomes
Ola amigo. Normalmente essas equações diofantinas nao lineares tem solução
passando por congruência.



Em qui., 11 de ago. de 2022 16:11, Esaú Gomes 
escreveu:

> Alguém poderia me falar o que estudar mais especificamente na questão
> abaixo?
>
> Para quais valores naturais  de *n* e *x*, existe solução
> 2^n = 3x + 1.
>
> --
> Esta mensagem foi verificada pelo sistema de antivírus e
> acredita-se estar livre de perigo.

-- 
Esta mensagem foi verificada pelo sistema de antiv�rus e
 acredita-se estar livre de perigo.



[obm-l] Fwd: Módulo

2022-08-11 Por tôpico Esaú Gomes
Alguém poderia me falar o que estudar mais especificamente na questão
abaixo?

Para quais valores naturais  de *n* e *x*, existe solução
2^n = 3x + 1.

-- 
Esta mensagem foi verificada pelo sistema de antiv�rus e
 acredita-se estar livre de perigo.



[obm-l] Re: [obm-l] Provar que a inteira f é um polinômio de grau positivo

2022-07-15 Por tôpico Artur Costa Steiner
Obrigado, abordagem bem interessante

Eu dei a seguinte prova:

Para z em C/{0}, seja g(z) = f(1/z), obtendo-se uma função holomorfa tal
que lim z —> 0 g(z) = lim z—> oo f(z) = oo. Assim, g é meromorfa em C,
tendo em 0 seu único polo. Sendo n > 0 a ordem deste polo, g é expandida em
C/{0} por uma série de Laurent em torno de 0, havendo portanto complexos
c(-n), … c(0), c(1) ….tais que

g(z) = Soma (k = -n, oo) c(k) z^k, z em C/{0}

Para z em  C/{0} temos então que

f(z) = g(1/z) =Soma (k = n, -oo c(k) z^(k) (1)

Em (1), temos a série de Laurent que, em C/{0}, expande f e em torno de 0.
Se nesta série houvesse algum coeficiente não nulo associado a potência
negativa de z, f apresentaria uma singularidade em 0. Mas sendo uma função
 inteira, f não apresenta nenhuma singularidade em C, do que deduzimos que,
em (1), todos os coeficientes associados a potências negativas de z são
nulos. Logo, em C/{0} f é o polinômio de grau n dado por

f(z) = c(-n) z^n + ….  c(1) z + c(0) (2)

Como f é contínua, temos que f(0) = lim z —> 0 f(z) = c(0), o que mostra
que (2) vale em todo o C. Logo, f é em C um polinômio de grau positivo.

Abs
Artur

Em qui., 14 de jul. de 2022 às 19:23, Claudio Buffara <
claudio.buff...@gmail.com> escreveu:

> Use o fato de que toda função meromorfica  em C união {inf} é da forma
> f(z)/g(z), onde f, g são polinômios.
> Daí, como a função do enunciado é inteira, g(z) é constante (e não nula).
> E como f(z) rende a inf quando z tende a inf, f é um polinômio não
> constante.
>
> Enviado do meu iPhone
>
> > Em 14 de jul. de 2022, à(s) 16:41, Artur Costa Steiner <
> artur.costa.stei...@gmail.com> escreveu:
> >
> > Oi amigos!
> >
> > Um teorema da Análise Complexa diz que, se f é inteira e lim z —> oo
> f(z) = oo, então f é um polinômio (claramente não constante). Nos
> livros em que estudei isso era dado como exercício, de modo que nunca vi a
> demonstração deste teorema. Eu consegui dar duas demonstrações para
> ele, sendo que uma delas sei que está certa A outra acho que também está
> certa, mas a primeira me parece bem melhor.Â
> >
> > Alguém aqui pode dar uma prova, para comparar com a minha? Se houver
> interesse (Análise Complexa não costuma aparecer aqui) eu dou as minhas.Â
> >
> > Obrigado
> >
> > Artur
> >
> >
> >
> > --
> > Esta mensagem foi verificada pelo sistema de antivírus e
> > acredita-se estar livre de perigo.
>
> --
> Esta mensagem foi verificada pelo sistema de antivírus e
>  acredita-se estar livre de perigo.
>
>
> =
> Instru�ões para entrar na lista, sair da lista e usar a lista em
> http://www.mat.puc-rio.br/~obmlistas/obm-l.html
> =
>

-- 
Esta mensagem foi verificada pelo sistema de antiv�rus e
 acredita-se estar livre de perigo.



[obm-l] Re: [obm-l] Função phi de Euler

2022-07-15 Por tôpico Anderson Torres
Em qui, 14 de jul de 2022 11:52, Rubens Vilhena Fonseca <
rubens.vilhen...@gmail.com> escreveu:

> Saudações a todos da lista.
> É um fato que para primos p ímpares, a função de Euler phi(p)=p-1 é sempre
> um valor par.
> Os primos 7, 13, 19, 31, 37, 67, 73, 79, 97, ... tem valores pares
> múltiplos de 3.
> Existe algum caminho a tomar para determinar quando phi(p) = 3 .(2k)?
>

quem é esse k?

Agradeço qualquer solução ou  informação ou indicação de leituras sobre o
> problema.
> Att
>
>
> --
> Esta mensagem foi verificada pelo sistema de antivírus e
> acredita-se estar livre de perigo.

-- 
Esta mensagem foi verificada pelo sistema de antiv�rus e
 acredita-se estar livre de perigo.



[obm-l] Re: [obm-l] Re: [obm-l] Função phi de Euler

2022-07-15 Por tôpico Anderson Torres
Em qui, 14 de jul de 2022 12:19, Esdras Muniz 
escreveu:

> Quis dizer φ(p)=p-1.
>
> Em qui, 14 de jul de 2022 12:02, Esdras Muniz 
> escreveu:
>
>> Oi(o)=p-1, aí isso só vale se o primo for da firma 6k+1.
>>
>
phi(4+3)=7-1


>> Em qui, 14 de jul de 2022 11:52, Rubens Vilhena Fonseca <
>> rubens.vilhen...@gmail.com> escreveu:
>>
>>> Saudações a todos da lista.
>>> É um fato que para primos p ímpares, a função de Euler phi(p)=p-1 é
>>> sempre um valor par.
>>> Os primos 7, 13, 19, 31, 37, 67, 73, 79, 97, ... tem valores pares
>>> múltiplos de 3.
>>> Existe algum caminho a tomar para determinar quando phi(p) = 3 .(2k)?
>>> Agradeço qualquer solução ou  informação ou indicação de leituras sobre
>>> o  problema.
>>> Att
>>>
>>>
>>> --
>>> Esta mensagem foi verificada pelo sistema de antivírus e
>>> acredita-se estar livre de perigo.
>>
>>
> --
> Esta mensagem foi verificada pelo sistema de antivírus e
> acredita-se estar livre de perigo.

-- 
Esta mensagem foi verificada pelo sistema de antiv�rus e
 acredita-se estar livre de perigo.



Re: [obm-l] Provar que a inteira f é um polinômio de grau positivo

2022-07-14 Por tôpico Claudio Buffara
Use o fato de que toda função meromorfica  em C união {inf} é da forma 
f(z)/g(z), onde f, g são polinômios.
Daí, como a função do enunciado é inteira, g(z) é constante (e não nula).
E como f(z) rende a inf quando z tende a inf, f é um polinômio não constante.

Enviado do meu iPhone

> Em 14 de jul. de 2022, à(s) 16:41, Artur Costa Steiner 
>  escreveu:
> 
> Oi amigos!
> 
> Um teorema da Análise Complexa diz que, se f é inteira e lim z —> oo f(z) 
> = oo, então f é um polinômio (claramente não constante). Nos livros em 
> que estudei isso era dado como exercício, de modo que nunca vi a 
> demonstração deste teorema. Eu consegui dar duas demonstrações para ele, 
> sendo que uma delas sei que está certa A outra acho que também está certa, 
> mas a primeira me parece bem melhor. 
> 
> Alguém aqui pode dar uma prova, para comparar com a minha? Se houver 
> interesse (Análise Complexa não costuma aparecer aqui) eu dou as minhas. 
> 
> Obrigado
> 
> Artur
> 
> 
> 
> -- 
> Esta mensagem foi verificada pelo sistema de antivírus e 
> acredita-se estar livre de perigo.

-- 
Esta mensagem foi verificada pelo sistema de antiv�rus e
 acredita-se estar livre de perigo.


=
Instru��es para entrar na lista, sair da lista e usar a lista em
http://www.mat.puc-rio.br/~obmlistas/obm-l.html
=


[obm-l] Provar que a inteira f é um polinômio de grau positivo

2022-07-14 Por tôpico Artur Costa Steiner
Oi amigos!

Um teorema da Análise Complexa diz que, se f é inteira e lim z —> oo f(z) =
oo, então f é um polinômio (claramente não constante). Nos livros em que
estudei isso era dado como exercício, de modo que nunca vi a demonstração
deste teorema. Eu consegui dar duas demonstrações para ele, sendo que uma
delas sei que está certa A outra acho que também está certa, mas a primeira
me parece bem melhor.

Alguém aqui pode dar uma prova, para comparar com a minha? Se houver
interesse (Análise Complexa não costuma aparecer aqui) eu dou as minhas.

Obrigado

Artur

-- 
Esta mensagem foi verificada pelo sistema de antiv�rus e
 acredita-se estar livre de perigo.



[obm-l] Re: [obm-l] Função phi de Euler

2022-07-14 Por tôpico Esdras Muniz
Quis dizer φ(p)=p-1.

Em qui, 14 de jul de 2022 12:02, Esdras Muniz 
escreveu:

> Oi(o)=p-1, aí isso só vale se o primo for da firma 6k+1.
>
> Em qui, 14 de jul de 2022 11:52, Rubens Vilhena Fonseca <
> rubens.vilhen...@gmail.com> escreveu:
>
>> Saudações a todos da lista.
>> É um fato que para primos p ímpares, a função de Euler phi(p)=p-1 é
>> sempre um valor par.
>> Os primos 7, 13, 19, 31, 37, 67, 73, 79, 97, ... tem valores pares
>> múltiplos de 3.
>> Existe algum caminho a tomar para determinar quando phi(p) = 3 .(2k)?
>> Agradeço qualquer solução ou  informação ou indicação de leituras sobre
>> o  problema.
>> Att
>>
>>
>> --
>> Esta mensagem foi verificada pelo sistema de antivírus e
>> acredita-se estar livre de perigo.
>
>

-- 
Esta mensagem foi verificada pelo sistema de antiv�rus e
 acredita-se estar livre de perigo.



[obm-l] Re: [obm-l] Função phi de Euler

2022-07-14 Por tôpico Esdras Muniz
Oi(o)=p-1, aí isso só vale se o primo for da firma 6k+1.

Em qui, 14 de jul de 2022 11:52, Rubens Vilhena Fonseca <
rubens.vilhen...@gmail.com> escreveu:

> Saudações a todos da lista.
> É um fato que para primos p ímpares, a função de Euler phi(p)=p-1 é sempre
> um valor par.
> Os primos 7, 13, 19, 31, 37, 67, 73, 79, 97, ... tem valores pares
> múltiplos de 3.
> Existe algum caminho a tomar para determinar quando phi(p) = 3 .(2k)?
> Agradeço qualquer solução ou  informação ou indicação de leituras sobre o
> problema.
> Att
>
>
> --
> Esta mensagem foi verificada pelo sistema de antivírus e
> acredita-se estar livre de perigo.

-- 
Esta mensagem foi verificada pelo sistema de antiv�rus e
 acredita-se estar livre de perigo.



[obm-l] Função phi de Euler

2022-07-14 Por tôpico Rubens Vilhena Fonseca
Saudações a todos da lista.
É um fato que para primos p ímpares, a função de Euler phi(p)=p-1 é sempre
um valor par.
Os primos 7, 13, 19, 31, 37, 67, 73, 79, 97, ... tem valores pares
múltiplos de 3.
Existe algum caminho a tomar para determinar quando phi(p) = 3 .(2k)?
Agradeço qualquer solução ou  informação ou indicação de leituras sobre o
problema.
Att

-- 
Esta mensagem foi verificada pelo sistema de antiv�rus e
 acredita-se estar livre de perigo.



Re: [obm-l] Ajuda em Repunits

2022-07-10 Por tôpico Rubens Vilhena Fonseca
Muito interessante, não faço a mínima ideia de como fazer, mas como você
disse vou me divertir pesquisando. Não sei se tem alguma coisa a ver mas,
se dividir o período desses exemplos ao "meio"  e somar (1/11 deu essa
ideia) o resultado parecem ser 9's. Outra coisa que percebi é que a ordem
desses denominadores módulo 10 é igual ao tamanho do período ( de novo 1/11
deu essa ideia). E como alguns são raízes primitivas de 10 o período é o
maior possível...
Com certeza se for verdade, são fatos já provados, vou tentar encontrar as
fontes.
Obrigado pela atenção
[[ ]]'s


Em dom., 10 de jul. de 2022 às 16:38, Claudio Buffara <
claudio.buff...@gmail.com> escreveu:

> Se quiser se divertir mais com isso, veja o seguinte:
> 1/7 = 0,142857142857142...
> O período é 142 857 e 1+8 = 4+5 = 2+7 = 9.
>
> 1/11: o período é 09 e 0+9 = 9.
>
> 1/13: o período é 076 923 e 0+9 = 7+2 = 6+3 = 9.
>
> Determine, com demonstração, para quais números N, o período de 1/N tem
> esta propriedade.
>
>
>
>
> On Sun, Jul 10, 2022 at 8:41 AM Rubens Vilhena Fonseca <
> rubens.vilhen...@gmail.com> wrote:
>
>> Muito obrigado ao Ralph Costa Teixeira e ao Claudio Buffara por todos os
>> ótimos esclarecimentos.
>> [[ ]]'s
>>
>> Em dom., 10 de jul. de 2022 às 01:39, Ralph Costa Teixeira <
>> ralp...@gmail.com> escreveu:
>>
>>> Argh, corrigindo um detalhe ali perto do fim:
>>> -- Sabemos que 10^q*B-B=r/10^w, portanto 9*(111...)**x**10^w = r*n.
>>> Novamente, como n é primo com 2, 3 e 5 *e x*, conclui-se que n divide
>>> 111 (com q 1's), e portanto q>=p=k.
>>>
>>> On Sun, Jul 10, 2022 at 1:24 AM Ralph Costa Teixeira 
>>> wrote:
>>>
 A chave: *os "restos parciais" que aparecem são exatamente os restos
 que x, 10x, 100x,  deixam na divisão por n.*
 ---///---

 MAIS SPOILERS ABAIXO


 ...


 


 ...


 

 Acho que facilita bastante pensar no "período" de 1/n de outro jeito:
 ---///---
 LEMA:
 (i) Dado n não divisível por 2 ou 5, existe algum número da forma
 111...111 que é múltiplo de n.
 (ii) Se n não for divisível por 2, 3 ou 5, o *menor* número do tipo
 111...111 que é múltiplo de n tem k dígitos, onde k é exatamente o tamanho
 do período (fundamental) da dízima em 1/n.
 PROVA:

 (i) Olhe os restos de 1, 11, 111, , ... na divisão por n. São n
 possibilidades, de 0 a n-1, então alguma hora algum resto tem que repetir.
 Isto significa que .. (com A dígitos) e 11...111 (com B dígitos,
 B>>> ...1110 (A 1's e B 0's) = 111 * (10^B) é múltiplo de n. Mas
 n não tem fator comum com aquele 10^B (pois não é divisível por 2 nem por
 5), portanto ...111 (com k=A-B dígitos) é divisível por n.

 (ii) Denote por P=111111 (com p dígitos) o menor daqueles caras com
 apenas "1s" que é múltiplo de n, e denote por k o "período fundamental" na
 dízima de 1/n.
 Por um lado, como 9P=999=10^p-1 é múltiplo de n, temos 10^p *
 (1/n) - 1/n inteiro. Mas isso significa que a parte decimal de 1/n "se
 repete" de p em p dígitos, ou seja, que a dízima de 1/n tem período p. Em
 particular, p>=k.
 Por outro lado, sendo k o período fundamental, temos 10^k * (1/n) -
 (1/n) com número finito de casas decimais, ou seja, (10^k-1)/n = m/10^z com
 m inteiro, e z=número de casas decimais que "sobraram". Mas daqui vem
 9*(111...111)*10^z = m*n (com k dígitos 1s). Como n é primo com 2, 3 e 5,
 conclui-se que 111... (k 1's) tem que ser múltiplo de n, e portanto
 k>=p.

 Note um efeito colateral disso tudo: provamos que 10^k*(1/n)- 1/n =
 10^p*(1/n)-1/n = inteiro. Assim aquele z vale 0, ou seja, não tem "casas
 decimais que sobram" -- a dízima periódica do 1/n se inicia logo no
 primeiro dígito!

 ---///---
 Agora fica tudo bem simples:
 a) Na notação acima, provamos que k=p, e n divide 111 com p
 dígitos.
 b) Seja q o período (fundamental) da dízima de B=x/n irredutível.

 Em primeiro lugar, provemos que q=k. Basicamente repetimos o que
 fizemos no lema:
 -- Sabemos que 10^q*B-B=r/10^w, portanto 9*(111...)*10^w = r*n.
 Novamente, como n é primo com 2, 3 e 5, conclui-se que n divide 111
 (com q 1's), e portanto q>=p=k.
 -- Por outro lado, como (10^k-1)/n é inteiro, (10^k-1)*x/n=10^k*B-B
 também é inteiro, ou seja, a dízima de B tem período k (e se inicia no
 primeiro dígito!). Portanto k>=q.

 *Enfim, note que os tais "restos parciais" que aparecem são exatamente
 os restos que x, 10x, 100x, , 10^q.x deixam na divisão por n. *A
 soma desses caras vale (...)*x, que é divisível por n pois temos
 ali q=k=p dígitos 1. Por isso, ao dividir esses restos parciais por n, a
 soma dos novos restos tem que ser múltiplo de n tambem.

 Foi?


 On Sat, Jul 9, 2022 at 7:16 PM Rubens Vilhena 

Re: [obm-l] Ajuda em Repunits

2022-07-10 Por tôpico Claudio Buffara
Se quiser se divertir mais com isso, veja o seguinte:
1/7 = 0,142857142857142...
O período é 142 857 e 1+8 = 4+5 = 2+7 = 9.

1/11: o período é 09 e 0+9 = 9.

1/13: o período é 076 923 e 0+9 = 7+2 = 6+3 = 9.

Determine, com demonstração, para quais números N, o período de 1/N tem
esta propriedade.




On Sun, Jul 10, 2022 at 8:41 AM Rubens Vilhena Fonseca <
rubens.vilhen...@gmail.com> wrote:

> Muito obrigado ao Ralph Costa Teixeira e ao Claudio Buffara por todos os
> ótimos esclarecimentos.
> [[ ]]'s
>
> Em dom., 10 de jul. de 2022 às 01:39, Ralph Costa Teixeira <
> ralp...@gmail.com> escreveu:
>
>> Argh, corrigindo um detalhe ali perto do fim:
>> -- Sabemos que 10^q*B-B=r/10^w, portanto 9*(111...)**x**10^w = r*n.
>> Novamente, como n é primo com 2, 3 e 5 *e x*, conclui-se que n divide
>> 111 (com q 1's), e portanto q>=p=k.
>>
>> On Sun, Jul 10, 2022 at 1:24 AM Ralph Costa Teixeira 
>> wrote:
>>
>>> A chave: *os "restos parciais" que aparecem são exatamente os restos
>>> que x, 10x, 100x,  deixam na divisão por n.*
>>> ---///---
>>>
>>> MAIS SPOILERS ABAIXO
>>>
>>>
>>> ...
>>>
>>>
>>> 
>>>
>>>
>>> ...
>>>
>>>
>>> 
>>>
>>> Acho que facilita bastante pensar no "período" de 1/n de outro jeito:
>>> ---///---
>>> LEMA:
>>> (i) Dado n não divisível por 2 ou 5, existe algum número da forma
>>> 111...111 que é múltiplo de n.
>>> (ii) Se n não for divisível por 2, 3 ou 5, o *menor* número do tipo
>>> 111...111 que é múltiplo de n tem k dígitos, onde k é exatamente o tamanho
>>> do período (fundamental) da dízima em 1/n.
>>> PROVA:
>>>
>>> (i) Olhe os restos de 1, 11, 111, , ... na divisão por n. São n
>>> possibilidades, de 0 a n-1, então alguma hora algum resto tem que repetir.
>>> Isto significa que .. (com A dígitos) e 11...111 (com B dígitos,
>>> B>> ...1110 (A 1's e B 0's) = 111 * (10^B) é múltiplo de n. Mas
>>> n não tem fator comum com aquele 10^B (pois não é divisível por 2 nem por
>>> 5), portanto ...111 (com k=A-B dígitos) é divisível por n.
>>>
>>> (ii) Denote por P=111111 (com p dígitos) o menor daqueles caras com
>>> apenas "1s" que é múltiplo de n, e denote por k o "período fundamental" na
>>> dízima de 1/n.
>>> Por um lado, como 9P=999=10^p-1 é múltiplo de n, temos 10^p *
>>> (1/n) - 1/n inteiro. Mas isso significa que a parte decimal de 1/n "se
>>> repete" de p em p dígitos, ou seja, que a dízima de 1/n tem período p. Em
>>> particular, p>=k.
>>> Por outro lado, sendo k o período fundamental, temos 10^k * (1/n) -
>>> (1/n) com número finito de casas decimais, ou seja, (10^k-1)/n = m/10^z com
>>> m inteiro, e z=número de casas decimais que "sobraram". Mas daqui vem
>>> 9*(111...111)*10^z = m*n (com k dígitos 1s). Como n é primo com 2, 3 e 5,
>>> conclui-se que 111... (k 1's) tem que ser múltiplo de n, e portanto
>>> k>=p.
>>>
>>> Note um efeito colateral disso tudo: provamos que 10^k*(1/n)- 1/n =
>>> 10^p*(1/n)-1/n = inteiro. Assim aquele z vale 0, ou seja, não tem "casas
>>> decimais que sobram" -- a dízima periódica do 1/n se inicia logo no
>>> primeiro dígito!
>>>
>>> ---///---
>>> Agora fica tudo bem simples:
>>> a) Na notação acima, provamos que k=p, e n divide 111 com p
>>> dígitos.
>>> b) Seja q o período (fundamental) da dízima de B=x/n irredutível.
>>>
>>> Em primeiro lugar, provemos que q=k. Basicamente repetimos o que fizemos
>>> no lema:
>>> -- Sabemos que 10^q*B-B=r/10^w, portanto 9*(111...)*10^w = r*n.
>>> Novamente, como n é primo com 2, 3 e 5, conclui-se que n divide 111
>>> (com q 1's), e portanto q>=p=k.
>>> -- Por outro lado, como (10^k-1)/n é inteiro, (10^k-1)*x/n=10^k*B-B
>>> também é inteiro, ou seja, a dízima de B tem período k (e se inicia no
>>> primeiro dígito!). Portanto k>=q.
>>>
>>> *Enfim, note que os tais "restos parciais" que aparecem são exatamente
>>> os restos que x, 10x, 100x, , 10^q.x deixam na divisão por n. *A
>>> soma desses caras vale (...)*x, que é divisível por n pois temos
>>> ali q=k=p dígitos 1. Por isso, ao dividir esses restos parciais por n, a
>>> soma dos novos restos tem que ser múltiplo de n tambem.
>>>
>>> Foi?
>>>
>>>
>>> On Sat, Jul 9, 2022 at 7:16 PM Rubens Vilhena Fonseca <
>>> rubens.vilhen...@gmail.com> wrote:
>>>
 Gostaria de uma demonstração para o seguinte teorema.
 *Teorema*. Seja n um inteiro positivo não divisível por 2, 3 ou 5, e
 suponha que a expansão decimal de l/n tenha período k. Então n é um fator
 do inteiro 111 ... 11 (k 1 's). Além disso, a soma dos restos parciais na
 divisão obtida de cada fração irredutível x/n é um múltiplo de n.
 Comentário:
 Pelo que entendi, se 1/13  tem período k =6. Então 13  divide 11 (
 k=6 1's).
 Essa parte consegui provar.
 Quanto à segunda parte  para 1/13 os resto da divisão sem repetição são
 {10, 9, 12, 3, 4, 1}. Então 10+9+12+3+4+1= 13q . (Não soube provar)
 Não consigo organizar uma sequência  de passos para a demonstração
 dos dois 

Re: [obm-l] Ajuda em Repunits

2022-07-10 Por tôpico Rubens Vilhena Fonseca
Muito obrigado ao Ralph Costa Teixeira e ao Claudio Buffara por todos os
ótimos esclarecimentos.
[[ ]]'s

Em dom., 10 de jul. de 2022 às 01:39, Ralph Costa Teixeira <
ralp...@gmail.com> escreveu:

> Argh, corrigindo um detalhe ali perto do fim:
> -- Sabemos que 10^q*B-B=r/10^w, portanto 9*(111...)**x**10^w = r*n.
> Novamente, como n é primo com 2, 3 e 5 *e x*, conclui-se que n divide
> 111 (com q 1's), e portanto q>=p=k.
>
> On Sun, Jul 10, 2022 at 1:24 AM Ralph Costa Teixeira 
> wrote:
>
>> A chave: *os "restos parciais" que aparecem são exatamente os restos que
>> x, 10x, 100x,  deixam na divisão por n.*
>> ---///---
>>
>> MAIS SPOILERS ABAIXO
>>
>>
>> ...
>>
>>
>> 
>>
>>
>> ...
>>
>>
>> 
>>
>> Acho que facilita bastante pensar no "período" de 1/n de outro jeito:
>> ---///---
>> LEMA:
>> (i) Dado n não divisível por 2 ou 5, existe algum número da forma
>> 111...111 que é múltiplo de n.
>> (ii) Se n não for divisível por 2, 3 ou 5, o *menor* número do tipo
>> 111...111 que é múltiplo de n tem k dígitos, onde k é exatamente o tamanho
>> do período (fundamental) da dízima em 1/n.
>> PROVA:
>>
>> (i) Olhe os restos de 1, 11, 111, , ... na divisão por n. São n
>> possibilidades, de 0 a n-1, então alguma hora algum resto tem que repetir.
>> Isto significa que .. (com A dígitos) e 11...111 (com B dígitos,
>> B> ...1110 (A 1's e B 0's) = 111 * (10^B) é múltiplo de n. Mas
>> n não tem fator comum com aquele 10^B (pois não é divisível por 2 nem por
>> 5), portanto ...111 (com k=A-B dígitos) é divisível por n.
>>
>> (ii) Denote por P=111111 (com p dígitos) o menor daqueles caras com
>> apenas "1s" que é múltiplo de n, e denote por k o "período fundamental" na
>> dízima de 1/n.
>> Por um lado, como 9P=999=10^p-1 é múltiplo de n, temos 10^p *
>> (1/n) - 1/n inteiro. Mas isso significa que a parte decimal de 1/n "se
>> repete" de p em p dígitos, ou seja, que a dízima de 1/n tem período p. Em
>> particular, p>=k.
>> Por outro lado, sendo k o período fundamental, temos 10^k * (1/n) - (1/n)
>> com número finito de casas decimais, ou seja, (10^k-1)/n = m/10^z com m
>> inteiro, e z=número de casas decimais que "sobraram". Mas daqui vem
>> 9*(111...111)*10^z = m*n (com k dígitos 1s). Como n é primo com 2, 3 e 5,
>> conclui-se que 111... (k 1's) tem que ser múltiplo de n, e portanto
>> k>=p.
>>
>> Note um efeito colateral disso tudo: provamos que 10^k*(1/n)- 1/n =
>> 10^p*(1/n)-1/n = inteiro. Assim aquele z vale 0, ou seja, não tem "casas
>> decimais que sobram" -- a dízima periódica do 1/n se inicia logo no
>> primeiro dígito!
>>
>> ---///---
>> Agora fica tudo bem simples:
>> a) Na notação acima, provamos que k=p, e n divide 111 com p
>> dígitos.
>> b) Seja q o período (fundamental) da dízima de B=x/n irredutível.
>>
>> Em primeiro lugar, provemos que q=k. Basicamente repetimos o que fizemos
>> no lema:
>> -- Sabemos que 10^q*B-B=r/10^w, portanto 9*(111...)*10^w = r*n.
>> Novamente, como n é primo com 2, 3 e 5, conclui-se que n divide 111
>> (com q 1's), e portanto q>=p=k.
>> -- Por outro lado, como (10^k-1)/n é inteiro, (10^k-1)*x/n=10^k*B-B
>> também é inteiro, ou seja, a dízima de B tem período k (e se inicia no
>> primeiro dígito!). Portanto k>=q.
>>
>> *Enfim, note que os tais "restos parciais" que aparecem são exatamente os
>> restos que x, 10x, 100x, , 10^q.x deixam na divisão por n. *A soma
>> desses caras vale (...)*x, que é divisível por n pois temos ali
>> q=k=p dígitos 1. Por isso, ao dividir esses restos parciais por n, a soma
>> dos novos restos tem que ser múltiplo de n tambem.
>>
>> Foi?
>>
>>
>> On Sat, Jul 9, 2022 at 7:16 PM Rubens Vilhena Fonseca <
>> rubens.vilhen...@gmail.com> wrote:
>>
>>> Gostaria de uma demonstração para o seguinte teorema.
>>> *Teorema*. Seja n um inteiro positivo não divisível por 2, 3 ou 5, e
>>> suponha que a expansão decimal de l/n tenha período k. Então n é um fator
>>> do inteiro 111 ... 11 (k 1 's). Além disso, a soma dos restos parciais na
>>> divisão obtida de cada fração irredutível x/n é um múltiplo de n.
>>> Comentário:
>>> Pelo que entendi, se 1/13  tem período k =6. Então 13  divide 11 (
>>> k=6 1's).
>>> Essa parte consegui provar.
>>> Quanto à segunda parte  para 1/13 os resto da divisão sem repetição são
>>> {10, 9, 12, 3, 4, 1}. Então 10+9+12+3+4+1= 13q . (Não soube provar)
>>> Não consigo organizar uma sequência  de passos para a demonstração
>>> dos dois fatos.
>>> Agradeço qualquer ajuda.
>>> [[ ]]'s
>>>
>>> --
>>> Esta mensagem foi verificada pelo sistema de antivírus e
>>> acredita-se estar livre de perigo.
>>
>>
> --
> Esta mensagem foi verificada pelo sistema de antivírus e
> acredita-se estar livre de perigo.

-- 
Esta mensagem foi verificada pelo sistema de antiv�rus e
 acredita-se estar livre de perigo.



Re: [obm-l] Ajuda em Repunits

2022-07-09 Por tôpico Claudio Buffara
Se n não é divisível por 2 e nem por 5, então 1/n = 0,a1a2...ak a1a2...ak
a1...  (dízima periódica simples de período k)
Daí (10^k)*n - n = a1a2...ak ==> (99...9)*n é inteiro (onde há k algarismos
9) ==> n é fator de 99...9 = 9*(11...1).
Mas n é primo com 3 ==> n | 11...1

Pra segunda parte, a ideia é tentar ver porque é verdade com exemplos
concretos.
Por exemplo, 1/7:
10*1 = 1*7 + 3
10*3 = 4*7 + 2
10*2 = 2*7 + 6
10*6 = 8*7 + 4
10*4 = 5*7 + 5
10*5 = 7*7 + 1
10*1 = 1*7 + 3  (e as equações se repetem a partir daqui)

1/13:
10*1 = 0*13 + 10
10*10 = 7*13 + 9
10*9 = 6*13 + 12
10*12 = 9*13 + 3
10*3 = 2*13 + 4
10*4 = 3*13 + 1
10*1 = 0*13 + 10 (idem)

Assim, no caso geral, pra calcular a representação de 1/n, as k primeiras
divisões sucessivas resultam em:
10*1 = a1*n + r1
10*r1 = a2*n + r2
10*r2 = a3*n + r3
...
10*r(k-1) = ak*n + rk

Como n é primo com 2 e 5, 1/n será uma dízima periódica simples, digamos de
período k.
Isso significa que rk, o resto da k-ésima divisão, será necessariamente
igual a 1, já que os dividendos (os algarismos aj que formam o período)
irão se repetir a partir da (k+1)-ésima equação.
Ou seja, a(k+1) = a1 e, portanto, r(k+1) = r1.

Somando as k equações, obtemos:
10*(1+r1+r2+ ...r(k-1)) = (a1+a2+a3...+ak)*n + (r1+r2+r3+...+rk).
Como rk = 1, isso fica:
10*(rk+r1+r2+ ...r(k-1)) = (a1+a2+a3...+ak)*n + (r1+r2+r3+...+rk) ==>
9*(rk+r2+...+r(k-1)) = (a1+a2+a3+...+ak)*n
Como n é primo com 3 (e, portanto, com 9), concluímos que n divide
r1+r2+...+rk.




On Sat, Jul 9, 2022 at 7:16 PM Rubens Vilhena Fonseca <
rubens.vilhen...@gmail.com> wrote:

> Gostaria de uma demonstração para o seguinte teorema.
> *Teorema*. Seja n um inteiro positivo não divisível por 2, 3 ou 5, e
> suponha que a expansão decimal de l/n tenha período k. Então n é um fator
> do inteiro 111 ... 11 (k 1 's). Além disso, a soma dos restos parciais na
> divisão obtida de cada fração irredutível x/n é um múltiplo de n.
> Comentário:
> Pelo que entendi, se 1/13  tem período k =6. Então 13  divide 11 ( k=6
> 1's).
> Essa parte consegui provar.
> Quanto à segunda parte  para 1/13 os resto da divisão sem repetição são
> {10, 9, 12, 3, 4, 1}. Então 10+9+12+3+4+1= 13q . (Não soube provar)
> Não consigo organizar uma sequência  de passos para a demonstração
> dos dois fatos.
> Agradeço qualquer ajuda.
> [[ ]]'s
>
> --
> Esta mensagem foi verificada pelo sistema de antivírus e
> acredita-se estar livre de perigo.

-- 
Esta mensagem foi verificada pelo sistema de antiv�rus e
 acredita-se estar livre de perigo.



Re: [obm-l] Ajuda em Repunits

2022-07-09 Por tôpico Ralph Costa Teixeira
Argh, corrigindo um detalhe ali perto do fim:
-- Sabemos que 10^q*B-B=r/10^w, portanto 9*(111...)**x**10^w = r*n.
Novamente, como n é primo com 2, 3 e 5 *e x*, conclui-se que n divide
111 (com q 1's), e portanto q>=p=k.

On Sun, Jul 10, 2022 at 1:24 AM Ralph Costa Teixeira 
wrote:

> A chave: *os "restos parciais" que aparecem são exatamente os restos que
> x, 10x, 100x,  deixam na divisão por n.*
> ---///---
>
> MAIS SPOILERS ABAIXO
>
>
> ...
>
>
> 
>
>
> ...
>
>
> 
>
> Acho que facilita bastante pensar no "período" de 1/n de outro jeito:
> ---///---
> LEMA:
> (i) Dado n não divisível por 2 ou 5, existe algum número da forma
> 111...111 que é múltiplo de n.
> (ii) Se n não for divisível por 2, 3 ou 5, o *menor* número do tipo
> 111...111 que é múltiplo de n tem k dígitos, onde k é exatamente o tamanho
> do período (fundamental) da dízima em 1/n.
> PROVA:
>
> (i) Olhe os restos de 1, 11, 111, , ... na divisão por n. São n
> possibilidades, de 0 a n-1, então alguma hora algum resto tem que repetir.
> Isto significa que .. (com A dígitos) e 11...111 (com B dígitos,
> B ...1110 (A 1's e B 0's) = 111 * (10^B) é múltiplo de n. Mas
> n não tem fator comum com aquele 10^B (pois não é divisível por 2 nem por
> 5), portanto ...111 (com k=A-B dígitos) é divisível por n.
>
> (ii) Denote por P=111111 (com p dígitos) o menor daqueles caras com
> apenas "1s" que é múltiplo de n, e denote por k o "período fundamental" na
> dízima de 1/n.
> Por um lado, como 9P=999=10^p-1 é múltiplo de n, temos 10^p *
> (1/n) - 1/n inteiro. Mas isso significa que a parte decimal de 1/n "se
> repete" de p em p dígitos, ou seja, que a dízima de 1/n tem período p. Em
> particular, p>=k.
> Por outro lado, sendo k o período fundamental, temos 10^k * (1/n) - (1/n)
> com número finito de casas decimais, ou seja, (10^k-1)/n = m/10^z com m
> inteiro, e z=número de casas decimais que "sobraram". Mas daqui vem
> 9*(111...111)*10^z = m*n (com k dígitos 1s). Como n é primo com 2, 3 e 5,
> conclui-se que 111... (k 1's) tem que ser múltiplo de n, e portanto
> k>=p.
>
> Note um efeito colateral disso tudo: provamos que 10^k*(1/n)- 1/n =
> 10^p*(1/n)-1/n = inteiro. Assim aquele z vale 0, ou seja, não tem "casas
> decimais que sobram" -- a dízima periódica do 1/n se inicia logo no
> primeiro dígito!
>
> ---///---
> Agora fica tudo bem simples:
> a) Na notação acima, provamos que k=p, e n divide 111 com p
> dígitos.
> b) Seja q o período (fundamental) da dízima de B=x/n irredutível.
>
> Em primeiro lugar, provemos que q=k. Basicamente repetimos o que fizemos
> no lema:
> -- Sabemos que 10^q*B-B=r/10^w, portanto 9*(111...)*10^w = r*n.
> Novamente, como n é primo com 2, 3 e 5, conclui-se que n divide 111
> (com q 1's), e portanto q>=p=k.
> -- Por outro lado, como (10^k-1)/n é inteiro, (10^k-1)*x/n=10^k*B-B também
> é inteiro, ou seja, a dízima de B tem período k (e se inicia no primeiro
> dígito!). Portanto k>=q.
>
> *Enfim, note que os tais "restos parciais" que aparecem são exatamente os
> restos que x, 10x, 100x, , 10^q.x deixam na divisão por n. *A soma
> desses caras vale (...)*x, que é divisível por n pois temos ali
> q=k=p dígitos 1. Por isso, ao dividir esses restos parciais por n, a soma
> dos novos restos tem que ser múltiplo de n tambem.
>
> Foi?
>
>
> On Sat, Jul 9, 2022 at 7:16 PM Rubens Vilhena Fonseca <
> rubens.vilhen...@gmail.com> wrote:
>
>> Gostaria de uma demonstração para o seguinte teorema.
>> *Teorema*. Seja n um inteiro positivo não divisível por 2, 3 ou 5, e
>> suponha que a expansão decimal de l/n tenha período k. Então n é um fator
>> do inteiro 111 ... 11 (k 1 's). Além disso, a soma dos restos parciais na
>> divisão obtida de cada fração irredutível x/n é um múltiplo de n.
>> Comentário:
>> Pelo que entendi, se 1/13  tem período k =6. Então 13  divide 11 (
>> k=6 1's).
>> Essa parte consegui provar.
>> Quanto à segunda parte  para 1/13 os resto da divisão sem repetição são
>> {10, 9, 12, 3, 4, 1}. Então 10+9+12+3+4+1= 13q . (Não soube provar)
>> Não consigo organizar uma sequência  de passos para a demonstração
>> dos dois fatos.
>> Agradeço qualquer ajuda.
>> [[ ]]'s
>>
>> --
>> Esta mensagem foi verificada pelo sistema de antivírus e
>> acredita-se estar livre de perigo.
>
>

-- 
Esta mensagem foi verificada pelo sistema de antiv�rus e
 acredita-se estar livre de perigo.



Re: [obm-l] Ajuda em Repunits

2022-07-09 Por tôpico Ralph Costa Teixeira
A chave: *os "restos parciais" que aparecem são exatamente os restos que x,
10x, 100x,  deixam na divisão por n.*
---///---

MAIS SPOILERS ABAIXO


...





...




Acho que facilita bastante pensar no "período" de 1/n de outro jeito:
---///---
LEMA:
(i) Dado n não divisível por 2 ou 5, existe algum número da forma 111...111
que é múltiplo de n.
(ii) Se n não for divisível por 2, 3 ou 5, o *menor* número do tipo
111...111 que é múltiplo de n tem k dígitos, onde k é exatamente o tamanho
do período (fundamental) da dízima em 1/n.
PROVA:

(i) Olhe os restos de 1, 11, 111, , ... na divisão por n. São n
possibilidades, de 0 a n-1, então alguma hora algum resto tem que repetir.
Isto significa que .. (com A dígitos) e 11...111 (com B dígitos,
B=k.
Por outro lado, sendo k o período fundamental, temos 10^k * (1/n) - (1/n)
com número finito de casas decimais, ou seja, (10^k-1)/n = m/10^z com m
inteiro, e z=número de casas decimais que "sobraram". Mas daqui vem
9*(111...111)*10^z = m*n (com k dígitos 1s). Como n é primo com 2, 3 e 5,
conclui-se que 111... (k 1's) tem que ser múltiplo de n, e portanto
k>=p.

Note um efeito colateral disso tudo: provamos que 10^k*(1/n)- 1/n =
10^p*(1/n)-1/n = inteiro. Assim aquele z vale 0, ou seja, não tem "casas
decimais que sobram" -- a dízima periódica do 1/n se inicia logo no
primeiro dígito!

---///---
Agora fica tudo bem simples:
a) Na notação acima, provamos que k=p, e n divide 111 com p dígitos.
b) Seja q o período (fundamental) da dízima de B=x/n irredutível.

Em primeiro lugar, provemos que q=k. Basicamente repetimos o que fizemos no
lema:
-- Sabemos que 10^q*B-B=r/10^w, portanto 9*(111...)*10^w = r*n.
Novamente, como n é primo com 2, 3 e 5, conclui-se que n divide 111
(com q 1's), e portanto q>=p=k.
-- Por outro lado, como (10^k-1)/n é inteiro, (10^k-1)*x/n=10^k*B-B também
é inteiro, ou seja, a dízima de B tem período k (e se inicia no primeiro
dígito!). Portanto k>=q.

*Enfim, note que os tais "restos parciais" que aparecem são exatamente os
restos que x, 10x, 100x, , 10^q.x deixam na divisão por n. *A soma
desses caras vale (...)*x, que é divisível por n pois temos ali
q=k=p dígitos 1. Por isso, ao dividir esses restos parciais por n, a soma
dos novos restos tem que ser múltiplo de n tambem.

Foi?


On Sat, Jul 9, 2022 at 7:16 PM Rubens Vilhena Fonseca <
rubens.vilhen...@gmail.com> wrote:

> Gostaria de uma demonstração para o seguinte teorema.
> *Teorema*. Seja n um inteiro positivo não divisível por 2, 3 ou 5, e
> suponha que a expansão decimal de l/n tenha período k. Então n é um fator
> do inteiro 111 ... 11 (k 1 's). Além disso, a soma dos restos parciais na
> divisão obtida de cada fração irredutível x/n é um múltiplo de n.
> Comentário:
> Pelo que entendi, se 1/13  tem período k =6. Então 13  divide 11 ( k=6
> 1's).
> Essa parte consegui provar.
> Quanto à segunda parte  para 1/13 os resto da divisão sem repetição são
> {10, 9, 12, 3, 4, 1}. Então 10+9+12+3+4+1= 13q . (Não soube provar)
> Não consigo organizar uma sequência  de passos para a demonstração
> dos dois fatos.
> Agradeço qualquer ajuda.
> [[ ]]'s
>
> --
> Esta mensagem foi verificada pelo sistema de antivírus e
> acredita-se estar livre de perigo.

-- 
Esta mensagem foi verificada pelo sistema de antiv�rus e
 acredita-se estar livre de perigo.



[obm-l] Ajuda em Repunits

2022-07-09 Por tôpico Rubens Vilhena Fonseca
Gostaria de uma demonstração para o seguinte teorema.
*Teorema*. Seja n um inteiro positivo não divisível por 2, 3 ou 5, e
suponha que a expansão decimal de l/n tenha período k. Então n é um fator
do inteiro 111 ... 11 (k 1 's). Além disso, a soma dos restos parciais na
divisão obtida de cada fração irredutível x/n é um múltiplo de n.
Comentário:
Pelo que entendi, se 1/13  tem período k =6. Então 13  divide 11 ( k=6
1's).
Essa parte consegui provar.
Quanto à segunda parte  para 1/13 os resto da divisão sem repetição são
{10, 9, 12, 3, 4, 1}. Então 10+9+12+3+4+1= 13q . (Não soube provar)
Não consigo organizar uma sequência  de passos para a demonstração dos dois
fatos.
Agradeço qualquer ajuda.
[[ ]]'s

-- 
Esta mensagem foi verificada pelo sistema de antiv�rus e
 acredita-se estar livre de perigo.



[obm-l] Re: [obm-l] Questão de probabilidade

2022-06-29 Por tôpico Rogerio Ponce
Ola' Vanderlei e pessoal da lista!
Pediram-me para resolver o problema por inteiro.
Ok, vamos la'!


Em um pet shop ha' 3 gatos e 5 caes. Sabemos que 3 desses animais sao
pretos, 4 sao brancos e 1 e' malhado. Alem disso, pelo menos 1
cachorro e' preto. Assinale o que for correto.
01) A probabilidade de haver exatamente 1 cachorro preto e' de 1/6.
02) A probabilidade de haver pelo menos 1 gato branco e pelo menos 2
cachorros brancos e' de 2/3.
04) A probabilidade de haver um cachorro malhado e' maior do que a
probabilidade de haver um gato malhado.
08) Se um animal for escolhido ao acaso, a probabilidade de ele ser um
cachorro preto e' de 1/8.
16) Se um animal for escolhido ao acaso, a probabilidade de ele ser um
gato malhado e' de 1/16.



Afirmacao 01) A probabilidade de haver exatamente 1 cachorro preto e' de 1/6.

-

O cachorro que ja' e' preto, deixaremos de lado, de modo que temos
3Gatos,4Caes para pintar com 2Preto,4Branco,1Malhado.
Bem, dizer que existe exatamente 1 cachorro preto, significa dizer que
as outras 2 pinturas pretas foram para os gatos, ou que existem
exatamente 2 gatos pretos.

Entao, vamos comecar a pintar com a tinta preta, e verificar de
quantas formas podemos fazer isso.

Repare que para ponderar corretamente as ocorrencias que interessam,
eu considero todas as permutacoes possiveis, ou das tintas, ou dos
animais. Assim, posso, por exemplo, fixar a ordem das tintas, e
aplicar isso em todas as permutacoes possiveis entre os animais.
A razao entre os casos que interessam e os casos possiveis e'
justamente a probabilidade de ocorrencia de um caso que interessa.

Como sao 2 pinturas pretas, temos um total de 7 escolhas de animal
para a primeira pintura, e 6 escolhas para a segunda, com um total de
42 escolhas para a primeira e segunda pinturas pretas.

Deste total, temos 3 gatos para a primeira escolha e 2 gatos para a
segunda, com um total de 6 escolhas para termos 2 gatos pretos.
Deste total, vamos escolher 1 entre 3 gatos para a primeira pintura, e
1 entre 2 gatos para a segunda pintura, com um total de 6 escolhas
para pintarmos 2 gatos de preto.

Assim, a probabilidade de termos exatamente 2 gatos pretos vale 6/42 = 1/7
Ou seja, a probabilidade de termos exatamente um cachorro preto vale
1/7, de modo que a afirmacao 01 esta' errada.

OBS: note que, entre os gatos g1, g2 e g3 (por exemplo), mesmo que
pintemos g1 e g2 de preto, a ordem e' importante, pois ordens
diferentes correspondem a permutacoes diferentes. Pintar de preto o
gato g1, e depois o gato g2, e' uma permutacao diferente daquela em
que pintamos de preto primeiramente o gato g2, e depois o gato g1,
embora o resultado final seja obtermos g1 e g2 pretos.



Afirmacao 02) A probabilidade de haver pelo menos 1 gato branco e pelo
menos 2 cachorros brancos e' de 2/3.

-

O cachorro que ja' e' preto pode ser deixado de lado.

Seguindo agora com as 4 pinturas brancas, temos um total de 7 escolhas
para o primeiro animal, 6 para o segundo, 5 para o terceiro, e 4 para
o quarto, de modo que existem 7*6*5*4 escolhas diferentes para as
pinturas brancas.

Entre os animais que receberao as pinturas brancas, existem os seguintes casos:

1o caso: 4 caes brancos
O numero de combinacoes de 4 caes entre 4 caes e' C(4,4)=1.
Assim, considerando-se as permutacoes entre estes 4 animais
brancos, obtemos um total de permutacoes diferentes igual a
C(4,4)*4! = 1*4! = 24

2o caso: 3 caes e 1 gato brancos
Como sao 3 caes de um total de 4 caes possiveis, e 1 gato de
um total de 3 gatos possiveis, o numero total de permutacoes vale
C(4,3) * C(3,1) * 4! = 4*3*4! = 288

3o caso: 2 caes e 2 gatos brancos
o numero total de permutacoes vale
C(4,2) * C(3,2) * 4! = 6*3*4! = 432

4o caso: 1 cao e 3 gatos brancos
o numero total de permutacoes vale
C(4,1) * C(3,3) * 4! = 4*1*4! = 96


Contabilizando-se o segundo e o terceiro casos (que sao os que nos
interessam), temos um total de (288+432)=720 permutacoes, de um
universo de 840 permutacoes, de modo que a probabilidade pedida vale
720/840 = 6/7

Portanto, a afirmacao 02 esta' errada.



Afirmacao 04) A probabilidade de haver um cachorro malhado e' maior do
que a probabilidade de haver um gato malhado.

-

Deixando de lado o cachorro que ja' e' preto, e seguindo agora com a
pintura malhada, temos 7 escolhas de animais, das quais 4 sao de
cachorros, e 3 sao de gatos.

Assim, a probabilidade de haver um cachorro malhado e' 4/7 , e a
probabilidade de haver um gato malhado e' 3/7.

Portanto, a afirmacao 04 esta' correta.



Afirmacao 08) Se um animal for escolhido ao acaso, a probabilidade de
ele ser um cachorro preto e' de 1/8.

-

Se houvesse apenas um cachorro preto - aquele que ja' seria mesmo
preto, por definicao - a probabilidade de escolhermos um 

<    1   2   3   4   5   6   7   8   9   10   >